MSK Flashcards

1
Q

Thumb extension nerve root

A

C8

How well did you know this?
1
Not at all
2
3
4
5
Perfectly
2
Q

Finger abduction nerve root

A

T1

How well did you know this?
1
Not at all
2
3
4
5
Perfectly
3
Q

Ankle dorsiflexion nerve root

A

L4

How well did you know this?
1
Not at all
2
3
4
5
Perfectly
4
Q

Big toe extension nerve root

A

L5

How well did you know this?
1
Not at all
2
3
4
5
Perfectly
5
Q

Ankle plantarflexion nerve root

A

S1

How well did you know this?
1
Not at all
2
3
4
5
Perfectly
6
Q

Knee flexion nerve root

A

S2

How well did you know this?
1
Not at all
2
3
4
5
Perfectly
7
Q

Elbow flexion and wrist extension nerve root

A

C6

How well did you know this?
1
Not at all
2
3
4
5
Perfectly
8
Q

Elbow extension and wrist flexion nerve root

A

C7

How well did you know this?
1
Not at all
2
3
4
5
Perfectly
9
Q

Scapula elevation nerve root

A

C4

How well did you know this?
1
Not at all
2
3
4
5
Perfectly
10
Q

Shoulder abduction nerve root

A

C5

How well did you know this?
1
Not at all
2
3
4
5
Perfectly
11
Q

A hyperirritable band of tight muscle and fascia that produces local numbness and tingling along with radiating pain is also known by which of the following names?

AMyalgia
BTrigger point
CTender point
DMyofascial pain

A

Answer: A

* A trigger point is defined as in the question stem. A trigger point can be palpated and appreciated by the physician. A tender point is something that cannot be truly felt by the physician, and does not typically radiate pain outward.
* Myofascial pain and myalgias are subjective feelings of pain arising from muscle + fascia, and muscle, respectively.
How well did you know this?
1
Not at all
2
3
4
5
Perfectly
12
Q

A 40 year-old male is water skiing when he develops sudden onset left knee pain after a wipeout. On exam, valgus stress testing is positive for laxity and reproduction of knee pain. MRI reveals a grade 3 medial collateral ligament tear in the knee (MCL tear). There is no evidence of avulsion fracture or other ligament tearing. Strength, sensation, and reflexes are otherwise intact. What is the next best step in management?

AKnee brace
BPhysical therapy
CSurgical MCL repair
DProlotherapy injection into the MCL

A

Answer: A
• MCL tears can be treated nonoperatively. A patient with advanced-grade tearing (as in this patient) should be treated with a knee brace to protect the MCL and facilitate healing, followed by gentle ROM and physical therapy.
• Surgery would be indicated if other complications were present such as other ligaments being torn, avulsion fractures, etc. Prolotherapy would not be appropriate for large ligament tears in general.

How well did you know this?
1
Not at all
2
3
4
5
Perfectly
13
Q

A patient with rheumatoid arthritis develops a swan neck deformity. How would you describe this abnormal finger position? DIP: distal interphalangeal joint. PIP: proximal interphalangeal joint. MCP: metacarpophalangeal joint.

ADIP flexion, PIP extension, MCP extension
BDIP extension, PIP flexion, MCP extension
CDIP extension, PIP flexion, MCP flexion
DDIP flexion, PIP extension, MCP flexion

A

Answer: D
“DIP flexion, PIP extension, MCP flexion” correctly describes a swan neck deformity. This is treated with a swan neck ring splint (imagine that!).

How well did you know this?
1
Not at all
2
3
4
5
Perfectly
14
Q

A 26 year-old male quarterback in a football game is tackled to the ground. He grabs his right ankle in pain. You rush onto the field and diagnose a torn tibiofibular interosseous ligament. X-rays demonstrate a Maisonneuve fracture. You advise the quarterback that:

APhysical therapy alone is recommended
BOrthopedic surgery consultation is recommended
CKnee bracing and rest will be sufficient
DHis playing days are over

A

Answer: B
• A Maisonneuve fracture is a proximal fibula fracture that may occur following a high ankle sprain (tibiofibular syndesmosis tear/rupture). Essentially the tearing of the syndesmosis extends all the way up proximally until the forces continue into the proximal fibula, causing a fracture there. X-rays of the knee are thus required following a high ankle sprain; Maisonneuve fracture generally requires orthopedic surgery consultation.

How well did you know this?
1
Not at all
2
3
4
5
Perfectly
15
Q

Which of the following nerves innervates the muscles of the deep posterior compartment of the leg?

A Tibial
B Sciatic
C Deep fibular
D Superficial fibular

A

Answer: A

Explanation:
• The deep posterior leg compartment contains the tibialis posterior, flexor digitorum longus, and flexor hallucis longus, which are all innervated by the tibial nerve.

How well did you know this?
1
Not at all
2
3
4
5
Perfectly
16
Q
Heterotopic ossification (HO) is most common at which of the following locations?
AHip
BKnee
CElbow
DShoulder
A

Answer: A
• In most patients, HO is most common at the hip.
• In burn patients, HO is most common at the elbow.

How well did you know this?
1
Not at all
2
3
4
5
Perfectly
17
Q
In evaluating a patient with pain, you decide to squeeze her left tibia together by wrapping your hands around the medial and lateral borders of the tibia and squeezing. The purpose of this test is to diagnose which of the following conditions?
AAchilles tendon rupture
BTibial plateau fracture
CMaisonneuve fracture
DShin splints
A

Answer: D
• Squeezing the tibia and reproducing a patient’s pain is indicative of shin splints (medial tibial stress syndrome).
• One may also perform a squeeze test to examine a high ankle sprain, but x-rays would be the best test for Maisonneuve fracture (a complication of a high ankle sprain), not physical exam.

How well did you know this?
1
Not at all
2
3
4
5
Perfectly
18
Q
A patient presents to your clinic with 5 months of right shoulder pain. On exam, you attempt passive abduction of the shoulder, which takes greater than usual force for you to abduct it, and the patient cries out in pain when you reach 70 degrees of abduction. Empty can, Neer, Hawkins, Speed, and O’Brien tests are positive. What is the most likely diagnosis?
ARotator cuff tear
BFrozen shoulder
CSubacromial impingement/bursitis
DLabral tear
A

Answer: B
• The key to answering this is the high difficulty in simply abducting the shoulder, as well as the limited degrees of abduction, which together imply a very tight shoulder capsule.
• Frozen shoulder (adhesive capsulitis) frequently shows pan-positive exam findings suggestive of other diseases.
• The other answer choices by themselves would not restrict shoulder range of motion to the same degree that frozen shoulder does.

How well did you know this?
1
Not at all
2
3
4
5
Perfectly
19
Q

A 6 year-old boy and his mother present to the ED with sudden-onset left elbow pain. The patient was on a walk with his mother. After his mother pulled him quickly away from the street where there were oncoming cars, he began to cry, and wouldn’t let his mother touch his left arm, holding it close to his body. What is the most likely underlying pathology?
ABehavioral etiology
BLateral epicondylitis
CSubluxation of the radial head away from the annular ligament
DPosterior elbow dislocation

A

Answer: C
• Nursemaid elbow is a subluxation of the radial head out of the annular ligament’s grasp. It is typically caused by a sudden axial force directed distally along the radius, which pulls the radial head out of the annular ligament’s grasp, causing the radial head to float anteriorly; this causes severe pain in the elbow.
• Treatment is physical exam, xrays, and closed reduction.
• Behavioral concerns should be the a diagnosis of exclusion after physical pathology has been ruled out. Lateral epicondylitis presents chronically.
• Posterior elbow dislocation typically occurs with trauma.

How well did you know this?
1
Not at all
2
3
4
5
Perfectly
20
Q

Which of the following is the function of the iliofemoral ligament regarding the hip?
ALimit abduction, flexion, internal rotation
BLimit abduction, extension, internal rotation
CLimit abduction, extension, external rotation
DLimit abduction, flexion, external rotation

A

Answer: C
• The iliofemoral ligament extends from the ileum to femur on the anterior side of the hip joint. Its function is to limit hip abduction, extension, and external rotation.

How well did you know this?
1
Not at all
2
3
4
5
Perfectly
21
Q
A 59 year-old female presents with left knee pain of gradual onset without a history of trauma. She points to the medial knee in a vague, large circle when you ask where the pain is. On exam strength, sensation, and reflexes are intact. Valgus stress testing is negative. Varus stress testing causes medial knee pain. There is pain with resisted hamstring strength. Palpation of the medial tibia below the knee elicits concordant pain. The left medial tibia appears swollen compared to the right. What is the most likely diagnosis?
APes anserine bursitis
BHamstring tendonitis
CLCL sprain
DMedial meniscus tear
A

Answer: A
• This patient depicts classic findings of pes anserine bursitis. The pes anserine consists of sartorius, gracilis, and semitendinosus which insert onto the medial tibia below the knee. There is also a bursa here which can become inflamed and distended.

How well did you know this?
1
Not at all
2
3
4
5
Perfectly
22
Q

A 27 year-old male attempts to cut to the right during a soccer game, and experiences sudden-onset left knee pain and swelling. Anterior drawer is negative. Lachman test is positive. X-rays of the knee should be taken to rule out a Segund fracture, which can be described as which of the following?

A Avulsion fracture of the medial tibial condyle
B Avulsion fracture of the lateral tibial condyle
C Avulsion fracture of the medial femoral condyle
D Avulsion fracture of the lateral femoral condyle

A

Answer: B

Explanation:
• ACL tears can be associated with Segund fractures, which is an avulsion fracture of the lateral tibial condyle. X-rays are important to rule out this fracture following an acute ACL tear.

How well did you know this?
1
Not at all
2
3
4
5
Perfectly
23
Q

ACL tears can be associated with Segund fractures, which is an avulsion fracture of the lateral tibial condyle. X-rays are important to rule out this fracture following an acute ACL tear. Which of the following maneuvers describes the Jerk test with respect to the shoulder?

ANone of these answers is correct
BFlexing shoulder beyond 90 degrees and applying posteroinferiorly directed axial force along humerus
CFlex to 90°, internal rotation, adduction across midline while applying posterior axial force
DAbducting, externally rotating, and applying anteriorly directed force on the humerus.

A

Answer: C
• Flexing the shoulder to 90 degrees, internally rotating, and adducting across midline with a posterior axial force describes the Jerk test, which is a test for posterior shoulder instability.
• The Kim test (flexing shoulder beyond 90 degrees and applying posteroinferior force) is also a shoulder posterior instability test.
• Abducting, externally rotating, and applying anterior force describes the anterior apprehension test, which unsurprisingly tests for anterior shoulder instability.

How well did you know this?
1
Not at all
2
3
4
5
Perfectly
24
Q

A 29 year-old male presents in follow up for left tennis elbow. He has tried RICE, PT, acetaminophen/NSAIDs, and corticosteroid injection to only minimal benefit. It is summer time, and he is disappointed because he is looking forward to playing in an upcoming tournament in London. He wonders if there is anything else he can do. Which of the following is the most appropriate recommendation?
A. Surgery
B. TENS
C. Needle tenotomy of common extensor tendon
D. Repeat corticosteroid injection

A

Answer: C
• This patient has failed conservative treatments as well as one corticosteroid injection which provided no benefit. At this point regenerative interventions may be considered, including needle tenotomy, platelet-rich plasma, prolotherapy, etc.
• Percutaneous needle tenotomy is the use of a needle to make small holes in a tendon through the skin. Repeated needlesticks can break up scar tissue and cause bleeding in a tendon, prompting the inflammatory cascade and helping the body’s own cells to begin rebuilding the tendon.

How well did you know this?
1
Not at all
2
3
4
5
Perfectly
25
Q
A 45 year-old male presents to your pain clinic with complaints of left elbow pain. On exam you note pain in the muscles and tendons just distal to the medial epicondyle. This patient’s condition is typically caused by which of the following
	A. Excessive varus forces
	B. Excellent sporting technique
	C. Trauma
	D. Overuse
A

Answer: D
• Golfer elbow (medial epicondylitis) is inflammation of the common flexor tendon of the elbow due to microtears/overuse of the wrist and finger flexors, which originate at the common flexor tendon off the medial epicondyle. It is typically caused by overuse and poor technique.

How well did you know this?
1
Not at all
2
3
4
5
Perfectly
26
Q
You are examining a patient from behind. Their stance has caught your attention. You notice that you can only see two of the patient’s left toes, but you can see all five of the patient’s right toes. What pathology do you suspect the most?
	A. Tibialis anterior insufficiency
	B. Tibialis posterior insufficiency
	C. Fibularis brevis insufficiency
	D. Fibularis longus insufficiency
A

Answer: B
• The “too many toes” sign is described here, caused by weakness/insufficiency of the tibialis posterior muscle/tendon unit.
• The tibialis posterior is a plantarflexor and invertor; thus, failure of this tendon to work properly allows the foot to be pulled pathologically into a hyperpronated position, causing more toes than usual to be observed from behind the patient (too many toes, indeed!).

How well did you know this?
1
Not at all
2
3
4
5
Perfectly
27
Q

A 65 year-old female is seen in follow-up for right shoulder pain. She has moderate to severe glenohumeral joint space narrowing on shoulder xray. She has completed a course of physical therapy and performs a home exercise program daily. Acetaminophen and ibuprofen provide mild benefit. She states that her function has improved but her pain remains in the shoulder. A corticosteroid injection you performed was of limited benefit. What is the next best step?

A Refer for spinal cord stimulator trial
B Start trial of oxycodone
C Orthopedic surgery referral
D Repeat corticosteroid injection

A

Answer: C

Explanation:
• This patient with advanced shoulder OA who has failed conservative treatments and injection should be considered for shoulder replacement via orthopedic surgery referral.
• Opioids are not indicated here. Repeat steroid injection has little utility here. Spinal cord stimulator trial would not be appropriate for this focal, structural source of pain.

How well did you know this?
1
Not at all
2
3
4
5
Perfectly
28
Q

A 29 year-old male presents to your general PM&R clinic with complaints of chronic bilateral shoulder pain. He denies weakness, numbness/tingling, and history of trauma. On exam, you note extreme hypermobility in multiple joints. Physical exam is grossly negative for reproduction of pain. What is the most appropriate treatment for this patient?

A There are no firm guidelines
B Physical therapy
C Surgery
D EMG

A

Answer: B

Explanation:
• This patient is suffering from bilateral shoulder dislocations, with a hint to that being the patient’s global hypermobility, which implies ligamentous laxity, which leads to the recurrent dislocations.
• Typically these patients lack trauma history, and their global hypermobility leads to both shoulders being bothered, rather than one, as typically is seen in traumatic unilateral shoulder dislocations.
• Bilateral shoulder dislocations due to hypermobility are most appropriately treated with rehab/physical therapy.
• Surgery may be considered failing therapy.

How well did you know this?
1
Not at all
2
3
4
5
Perfectly
29
Q

When deciding what type of work a patient is cleared to return to, “light duty work” is defined as lifting no more than how many lbs while on the job?

A 40
B 30
C 20
D 10

A

Answer: C

Explanation:
• Light duty is defined as no lifting over 20 lbs. Very light duty is lifting no more than 10 lbs.

How well did you know this?
1
Not at all
2
3
4
5
Perfectly
30
Q

The fifth extensor compartment of the wrist contains which of the following tendons?

A Extensor indicis proprius
B Extensor digiti minimi
C Extensor carpi ulnaris
D Extensor pollicis longus

A

Answer: B

Explanation:
• 1st extensor compartment contains Abductor pollicis longus and Extensor pollicis brevis tendons.
• 2nd extensor compartment contains Extensor carpi radialis longus and Extensor carpi radialis brevis tendons.
• 3rd extensor compartment contains Extensor pollicis longus tendon.
• 4th extensor compartment contains Extensor digitorum and Extensor indicis.
• 5th extensor compartment contains the Extensor digiti minimi tendon.

How well did you know this?
1
Not at all
2
3
4
5
Perfectly
31
Q

A patient with rheumatoid arthritis develops a Boutonniere deformity. How would you describe this abnormal finger position? DIP: distal interphalangeal joint. PIP: proximal interphalangeal joint. MCP: metacarpophalangeal joint.

A DIP flexion, PIP extension, MDP extension
B DIP extension, PIP flexion, MCP extension
C DIP extension, PIP flexion, MCP flexion
D DIP flexion, PIP extension, MCP flexion

A

Answer: B

Explanation:
• “DIP extension, PIP flexion, MCP extension” correctly describes a Boutonniere deformity. This is treated with a Boutonniere ring splint (imagine that!).

How well did you know this?
1
Not at all
2
3
4
5
Perfectly
32
Q

A 28 year-old male presents to your clinic with right ankle pain. You decide to stabilize the distal lower leg while grabbing the calcaneus and inverting the hindfoot. Which ligament are you attempting to test?

A Tibiofibular syndesmosis
B Posterior talofibular ligament
C Calcaneofibular ligament
D Anterior tibiofibular ligament

A

Answer: C

Explanation:
• Talar tilt test (described here) is a test for the ATFL (anterior talofibular ligament - NOT tibiofibular ligament as noted in this question) and the CFL (calcaneofibular ligament).
• It is used to define the extent of a lateral ankle sprain in terms of ligament involvement.

How well did you know this?
1
Not at all
2
3
4
5
Perfectly
33
Q

A 31 year-old female is involved in a skiing accident. She develops aching elbow pain shortly afterward. On exam you notice that, when extended, her elbow assumes a valgus angulation. Cozen test is weakly positive. When applying a laterally directed force across the elbow joint while stabilizing the arm and forearm, she cries out in pain. What is the most likely mechanism of injury in this patient?

A Excessive varus force across the elbow
B Excessive valgus force across the elbow
C Rupture of the common extensor tendon
D Overuse of the common extensor tendon

A

Answer: A

Explanation:
• This patient’s chief exam maneuver reproducing her pain foremost is the varus stress test, as demonstrated in the question stem.
• This test pulls apart the radial collateral ligament and causes pain if the ligament is already sprained or torn.
• Excessive varus forces across the elbow cause sprain of this ligament.
• Cozen being weakly positive is a red herring, as people in great pain will often demonstrate pan-positive exam findings.
• The purpose of Cozen’s test (also known as the “resisted wrist extension test” or “resistive tennis elbow test”) is to check for lateral epicondylalgia or “tennis elbow”.
○ The patient should be seated, with the elbow extended forearm maximal pronation, wrist radially abducted, and hand in a fist.
○ The therapist should stabilise elbow while palpating lateral epicondyle, other hand placed on the dorsum of the hand.
○ The patient is asked to move the wrist to dorsal flexion and the therapist provides resistance to this movement, in the position described above. The test is positive if pain on the lateral epicondyle is elicited.

How well did you know this?
1
Not at all
2
3
4
5
Perfectly
34
Q

A 22 year-old male sustains a lateral ankle sprain. Anterior drawer of the ankle, and talar tilt tests are positive. You grab your ultrasound machine because you are concerned about a secondary injury to which of the following structures most likely?

A Tibialis posterior tendon
B Tibialis anterior tendon
C Deltoid ligament
D Fibularis longus tendon

A

Answer: D

Explanation:
• A possible complication of lateral ankle sprains is a fibularis longus or brevis tendon injury.
• This is due to the fact that these tendons wrap around the lateral malleolus; thus, severe inversion forces can cause abrupt stretching and possible tearing of these tendons.
• Diagnostic ultrasound scan would be useful as an in-office, quick evaluation of the tendon anatomy.

How well did you know this?
1
Not at all
2
3
4
5
Perfectly
35
Q

A 20 year-old male is in a wrestling competition when he attempts to suplex his opponent by grabbing him about the waist and performing a deep squat while pivoting to his right. He experiences sudden-onset right knee pain afterward. Which of the following is the most likely diagnosis for the knee pain?

A MCL tear
B LCL tear
C Medial meniscus tear
D Lateral meniscus tear

A

Answer: D

Explanation:
• The important thing to learn in this question is the typical mechanism for meniscus injuries, which involves large axial forces on the knee (e.g. picking up another human being) while simultaneously performing a twisting/grinding motion upon the knee.

Further, know that deep squatting while pivoting under resistance is a classic mechanism for lateral meniscus tears.

How well did you know this?
1
Not at all
2
3
4
5
Perfectly
36
Q

A 21 year-old female long distance runner develops gradual onset bilateral medial shin pain. She is currently training for a marathon. She denies trauma. On exam, squeezing the medial and lateral tibia together reproduces her medial shin pain located along the tibia. What is the next best step?

A X-rays
B Physical therapy
C Reduce mileage
D Reassurance

A

Answer: A

Explanation:
• Bilateral shin pain in the context of long distance running, without trauma, is indicative of medial tibial stress syndrome (shin splints).
• However, a tibial stress fracture would present very similarly, and must be ruled out before the fracture line propagates into a larger fracture.
• Tibial x-rays are necessary.
• Relative rest, shoe orthotics, correct running gait (overpronation of the ankle is a common training error), physical therapy are all reasonable treatment options for shin splints (assuming x-rays are negative for stress fracture).

How well did you know this?
1
Not at all
2
3
4
5
Perfectly
37
Q

The tibialis posterior’s function is to do which of the following?

A Dorsiflexion and eversion
B Dorsiflexion and inversion
C Plantarflexion and eversion
D Plantarflexion and inversion

A

Answer: D

Explanation:
• The Tibialis posterior’s function is plantarflexion and inversion of the ankle.

How well did you know this?
1
Not at all
2
3
4
5
Perfectly
38
Q

For acutely inflamed joints in rheumatoid arthritis, which of the following is the most appropriate type of exercise?

A Isorheumatic
B Isometric
C Isotonic
D Isokinetic

A

Answer: B

Explanation:
• Isometric exercise is the best type of exercise for any acutely inflamed joint, as it limits the damage that can be done to the joint via range of motion (ROM) exercises.

How well did you know this?
1
Not at all
2
3
4
5
Perfectly
39
Q

A 37 year-old male is throwing the ‘ol pigskin (football) around to prove to his friends that he’s “still got it”. He is quickly tackled, and develops sudden-onset right ankle pain. With your priorities in line, you quickly point out that he has not in fact “still got it”. You then rush over and squeeze his lower leg with your two hands, which reproduces his pain. He does not appreciate that insult to injury and lets you know this. X-rays of the leg are normal. He says he wants to be treated like a high-level professional athlete who needs to return to the field as soon as possible. What is the next best step?

A Make non-weight-bearing in CAM boot
B Orthopedic surgery
C MRI
D Repeat x-ray in 1 week

A

Answer: C

Explanation:
• NWB status in a CAM boot is good treatment for high ankle sprains without bony fractures

However, MRI is indicated in cases of high suspicion for high ankle sprain with negative X-rays. The MRI would be important in order to define the extent of soft tissue injury in a high-level professional athlete.

How well did you know this?
1
Not at all
2
3
4
5
Perfectly
40
Q

A 17-year-old football player is the local high school’s star running back. In the 2nd game of the season, he attempts to make a cut on a planted right foot. He feels a ”pop” in his right knee and his leg gives out. He has immediate pain and swelling in his right knee and is unable to bear weight on that leg. MRI of the right knee the following day reveals a tear of his ACL (anterior cruciate ligament) and he undergoes surgical reconstruction. Which of the following orthoses is most appropriate post-operatively?

A Lenox-Hill derotation orthosis
B Unloader knee brace
C Swedish knee cage
D Hinged knee brace

A

Answer: A

Explanation:
• Following ACL reconstruction, a Lenox-Hill derotation orthosis is often used to control axial rotation of the knee as well as medial-lateral and anterior-posterior control
• Swedish knee cages are used for knee recurvatum to prevent knee hyperextension.
• Unloader braces are used most often for knee osteoarthritis; forces are “unloaded” from the more affected/symptomatic compartment, either medial or lateral; these forces are loaded onto the less symptomatic/arthritic side.
• Hinged knee braces offer increased stability and can be used for a wide array of knee pathologies.

How well did you know this?
1
Not at all
2
3
4
5
Perfectly
41
Q
  1. Which of the following muscles is not a wrist extensor?

A They are all wrist extensors
B Extensor digiti minimi
C Extensor indicis proprius
D Extensor digitorum

A

Answer: A

Explanation:
• All these muscles cross the wrist joint on the dorsal surface of the body; thus, they all provide some degree of wrist extension force.

How well did you know this?
1
Not at all
2
3
4
5
Perfectly
42
Q

A patient with a complete tear of the acromioclavicular (AC) ligament and partial tear of the coracoclavicular (CC) ligament has what grade of AC joint separation, and what should the treatment be?

A 3, surgery
B 3, rehab
C 2, surgery
D 2, rehab

A

Answer: D

Explanation:
• AC joint separations occur as grades 1-6.
• Grade 1: partial AC tear, intact CC, rehab.
• Grade 2: complete AC tear, partial CC tear, rehab.
• Grade 3: complete AC and CC tears, clavicle floats upward, rehab vs. surgery.
• Grade 4: complete AC and CC tears, clavicle floats superiorly and posteriorly, surgery.
• Grade 5: complete AC and CC tears, clavicle floats even more superiorly and posteriorly, surgery.
• Grade 6: complete AC and CC tears, clavicle floats downward, surgery.

How well did you know this?
1
Not at all
2
3
4
5
Perfectly
43
Q

A 29 year-old female presents with several months of widespread body pain. She has tried physical therapy to no benefit. She feels fatigued on most days. She is tender to palpation at multiple different locations throughout her body. You decide to recommend water aerobics and which other appropriate intervention?

A Duloxetine
B Tramadol
C TENS
D NSAIDs

A

Answer: A

Explanation:
• Fibromyalgia presents classically as this patient does. Generally no medical or imaging workup is required. There is often comorbid fatigue, anxiety, and depression, as well as lifestyle stressors.
• Aquatherapy/water aerobics is an excellent first-line treatment.
• SNRIs, SSRIs, TCAs, and gabapentin or pregabalin are also appropriate as medication trials.
• Generally a primary care physician or psychiatrist should handle an SSRI prescription.
• Opioids are NOT indicated in fibromyalgia.
• Duloxetine is an excellent initial pharmacologic therapy as a pain + mood medication.

“Standard” MSK-pain treatment modalities may not be effective in fibromyalgia, which carries a strong psychiatric component, hence Duloxetine’s appropriateness over TENS or NSAIDs.

How well did you know this?
1
Not at all
2
3
4
5
Perfectly
44
Q

A 68 year-old obese male presents with progressive, gradual onset right knee pain. He says the pain is “all over” the knee, and is associated with stiffness in the morning lasting 30 minutes. The pain is worse with weight-bearing and walking. Which of the following interventions will likely be the most impactful and appropriate for his knee pain?

A Orthopedic surgery consultation
B Knee corticosteroid injection
C Cane to offload the knee
D Weight loss

A

Answer: D

Explanation:
• Weight loss is the single most impactful intervention on knee OA pain due to obesity.
• Physical therapy should be combined with weight loss unless the patient is already at a healthy BMI.

How well did you know this?
1
Not at all
2
3
4
5
Perfectly
45
Q
In pronator teres syndrome, all median nerve-innervated muscles are affected EXCEPT which of the following?
AFlexor pollicis longus
BPronator quadratus
CFlexor carpi radialis
DPronator teres
A

Answer: D

Explanation:
• In PT (pronator teres) syndrome, the median nerve is compressed within the PT muscle, affecting all downstream median nerve-innervated muscles, except the PT (pronator teres) itself, which is actually innervated by a more proximal nerve branch that is not involved in the PT’s tight compression of the median nerve.
• All remaining median nerve and anterior interosseous nerve muscles will be affected, however (the remaining answer choices).

How well did you know this?
1
Not at all
2
3
4
5
Perfectly
46
Q
A patient with New York Heart Association (NYHA) class 3 heart failure is expected to be able to perform activities in which of the following MET (metabolic equivalent) ranges?
A5-7
B2-5
C2-3
D1-2
A

Answer: B

Explanation:
	• NYHA class 1 patients can perform activities over 7 METs.
	• NYHA class 2: anything between 5-7 METs.
	• NYHA class 3: anything between 2-5 METs.
	• NYHA class 4: there is dyspnea at rest.
How well did you know this?
1
Not at all
2
3
4
5
Perfectly
47
Q

A 23 year-old male presents with gradual onset 1 year of right knee pain. He is a division 1 basketball player and the national postseason tournament is about to begin. He is adamant that he must play, as his team is expected to win the championship. He has ibuprofen, tried 2 rounds of physical therapy and home exercises, as well as relative rest, all of which has not helped his pain. He has tenderness to palpation of his patellar tendon, and there is pain at the patellar tendon with resisted knee extension. His right patellar tendon appears swollen compared to his left. He has had unremarkable knee x-rays, and a knee MRI showed intact soft tissue structures. On diagnostic ultrasound scan, you note a thickened, hypoechoic, wavy appearance of the right patellar tendon compared to the fibrillar, relatively hyperechoic appearance of the left patellar tendon. Color doppler scan reveals punctate red and blue dots within the right patellar tendon, but not the left. What is the most appropriate next step?
ASit out the rest of the season and tournament
BUltrasound-guided patellar tendon scraping
COrthopedics consult
DUltrasound-guided corticosteroid injection

A

Answer: B

Explanation:
• This patient presents with classic findings of patellar tendonitis that has progressed into patellar tendonosis.
• Recall that tendonosis is a degenerative condition of a tendon that occurs due to tendon overuse. The tendon becomes wavy, thickened, and hypoechoic (dark) on ultrasound.
• Neurogenic inflammation accompanies this, in which neovessels and neonerves sprout from the Hoffa (infrapatellar) fat pad and extend into the patellar tendon, which causes chronic knee pain.
• Tendon scraping under ultrasound guidance involves guiding a needle between the patellar tendon and Hoffa fat pad (not into the tendon itself, despite the name “tendon scraping”) and moving the needle back and forth (superior to inferior and vice versa) in order to sever these neonerves and neovessels, thus treating the patient’s source of pain (the neonerves), and breaking the cycle of tendonosis, allowing the patient to progress in PT without the pain limitation from the neonerves.
• Orthopedic surgery is a last resort. Also, the patient may weight-bear immediately and perform physical activities after a tendon scraping, because the tendon itself was not pierced by a needle or scalpel, and so is still structurally intact.

How well did you know this?
1
Not at all
2
3
4
5
Perfectly
48
Q
The supraspinatus and deltoid contribute how many degrees to the function of shoulder abduction, respectively?
A165, 15
B15, 165
C150, 30
D30, 150
A

Answer: B

Explanation:
• The supraspinatus contributes to the first 15 degrees of shoulder abduction; beyond that point the deltoid tends to “take over”. This is due to both strength and mechanical/physical placement of these muscles.

How well did you know this?
1
Not at all
2
3
4
5
Perfectly
49
Q
All of the following are muscles of active inspiration except which of the following?
AInternal intercostals
BTrapezius
CLevator scapula
DScalenes
A

Answer: A

Explanation:
• Muscles of active inspiration include diaphragm, external intercostals, sternocleidomastoids, levator scapulae, scalenes, pectorals, and trapezius.
• Muscles of expiration include internal intercostals and abdominal muscles.

How well did you know this?
1
Not at all
2
3
4
5
Perfectly
50
Q
Which of the following muscles/tendons is not contained within the tarsal tunnel?
AFlexor digitorum brevis
BFlexor hallucis longus
CFlexor digitorum longus
DTibialis posterior
A

Answer: A

Explanation:
• Flexor digitorum brevis does NOT pass through the tarsal tunnel.

How well did you know this?
1
Not at all
2
3
4
5
Perfectly
51
Q
Which of the following is not a risk factor for the development of knee osteoarthritis (OA)?
ANSAID use
BWeak knee extensors
CWeak hip abductors
DHistory of ACL tear
A

Answer: A

Explanation:
• ACL tears and weak musculature controlling the knee and hip are risk factors for developing knee OA, as is obesity.

How well did you know this?
1
Not at all
2
3
4
5
Perfectly
52
Q

A 21 year-old male medical student presents to your clinic with complaints of anterior right knee pain for the past month. He is disappointed because he enjoys “wheeling and dealing” on the basketball court during competitive medical school basketball games. He denies significant trauma from basketball. On exam, resisted knee extension reproduces anterior knee pain. There is minimal tenderness to palpation of the soft tissue immediately superior to the patella. There is sharp tenderness to palpation of the soft tissue immediately inferior to the inferior pole of the patella. What is the most appropriate next step?
AKnee MRI to rule out tendon/ligament tear
BKnee injection
CPhysical therapy
DX-rays to rule out fracture

A

Answer: C

Explanation:
• This patient presents with classic patellar tendonitis, commonly associated with jumping sports (basketball, volleyball).
• Pain is located over the patellar tendon, generally where it inserts onto the patella, as described here.
• Treatment is relative rest, physical therapy, NSAIDs.
• There is no indication for imaging in this initial presentation of patellar tendonitis without trauma; this is a clinical diagnosis.

How well did you know this?
1
Not at all
2
3
4
5
Perfectly
53
Q

Kienbock Disease consists of which of the following pathologies, and what is the most appropriate treatment?
AIdiopathic avascular necrosis of the capitate; surgery
BIdiopathic avascular necrosis of the capitate; splinting
CIdiopathic avascular necrosis of the lunate; surgery
DIdiopathic avascular necrosis of the lunate; splinting

A

Answer: C

Explanation:
• Kienbock disease is another name for osteonecrosis of the lunate, which is idiopathic avascular necrosis of the lunate.
• Treatment is orthopedic surgery.

How well did you know this?
1
Not at all
2
3
4
5
Perfectly
54
Q
Regarding shoulder abduction range of motion, the number of degrees attributed to scapulothoracic motion compared to glenohumeral motion is which of the following?
A60:120
B130:50
C50:130
D90:90
A

Answer: A

Explanation:
• The 180 degrees of normal shoulder abduction ROM is due to 60 degrees of scapulothoracic ROM in combination with 120 degrees of glenohumeral range of motion.
• The key point here is this 2:1 ratio of glenohumeral ROM compared to scapulothoracic ROM, as well as knowing that abducting the shoulder is not purely because of movement of the ball within the socket, but because the scapula actually rotates upward to “help out” with this ROM; otherwise lots of subacromial shoulder impingement would occur.

How well did you know this?
1
Not at all
2
3
4
5
Perfectly
55
Q

Which of the following maneuvers describes the Jerk test with respect to the shoulder?
ANone of these answers is correct
BFlexing shoulder beyond 90 degrees and applying posteroinferiorly directed axial force along humerus
CFlex to 90°, internal rotation, adduction across midline while applying posterior axial force
DAbducting, externally rotating, and applying anteriorly directed force on the humerus

A

Answer: C

Explanation:
• Flexing the shoulder to 90 degrees, internally rotating, and adducting across midline with a posterior axial force describes the Jerk test, which is a test for posterior shoulder instability.
• The Kim test (flexing shoulder beyond 90 degrees and applying posteroinferior force) is also a shoulder posterior instability test.
• Abducting, externally rotating, and applying anterior force describes the anterior apprehension test, which unsurprisingly tests for anterior shoulder instability.

How well did you know this?
1
Not at all
2
3
4
5
Perfectly
56
Q
A 20 year-old male presents to your clinic with the chief complaint of right foot pain. The pain occurred gradually without trauma. On exam there is slight swelling proximal to the 3rd toe. Foot x-rays demonstrate a 3rd metatarsal stress fracture. This is otherwise known by what name?
AJones fracture
BCuboid fracture
CMarch fracture
DNutcracker fracture
A

Answer: C

Explanation:
• A March fracture is a metatarsal stress fracture.
• A Nutcracker fracture is a cuboid fracture and vice versa.
• A Jones fracture is a fracture across the base of the 5th metatarsal.

How well did you know this?
1
Not at all
2
3
4
5
Perfectly
57
Q
A 34 year-old male presents to your clinic for the evaluation of left periscapular pain. He has a history of motorcycle accident 1 year ago and has had this pain ever since then. On exam, you note his left scapula is positioned further laterally than the right scapula. Which nerve is implicated in this condition?
AUpper subscapular nerve
BLong thoracic nerve
CSpinal accessory nerve
DThoracodorsal nerve
A

Answer: C

Explanation:
• With a laterally winged scapula, the spinal accessory nerve (retracts) commanding the trapezius is deficient.
• With a medially winged scapula, the long thoracic nerve (protracts) commanding the serratus anterior would be deficient.
• The thoracodorsal nerve innervates the latissimus dorsi and does NOT cause a winged scapula when injured.
• The upper subscapular nerve innervates the subscapularis and teres major and does NOT cause a winged scapula when injured.

How well did you know this?
1
Not at all
2
3
4
5
Perfectly
58
Q
Which of the following is the most normal/ideal synovial fluid white blood cell count (WBC)?
A1000
B100
C50
D0
A

Answer: D

Explanation:
• Normal synovial fluid WBC should be zero (0) or nearly zero.

How well did you know this?
1
Not at all
2
3
4
5
Perfectly
59
Q

A 33 year-old male presents with right ankle pain and heel pain over the achilles tendon. He also endorses pain with urination. Which of the following etiologies is most likely responsible?

A Chlamydia
B Calcium pyrophosphate deposition
C Uric acid deposition
D Neisseria gonorrhea

A

Answer:

Explanation:
• Reactive Arthritis presents with asymmetric arthritis affecting especially the feet and ankles as well as enthesitis (e.g. achilles tendon pain).
• It also presents with urethritis and uveitis.
• This constellation of symptoms is “reactive” to an infection, typically chlamydia, campylobacter, or salmonella.
• Treatment involves antibiotics, NSAIDs, and physical therapy.

How well did you know this?
1
Not at all
2
3
4
5
Perfectly
60
Q
  1. You are performing a physical exam on a patient with knee pain. You lie the patient prone, flex their knee 90 degrees, and apply an axial force upon the patient’s heel (force directed superiorly from the heel into the calcaneus and tibia). What is the name of this test?

A McMurray
B Apley
C Thessaly
D D’Angelo

A

Answer: B

Explanation:
• The Apley Grind test is described here, and is a test for medial or lateral meniscus tears in the knee.
• Apley’s Distraction & Apley’s Gind tests:
○ If rotation plus distraction is more painful or shows increased rotation relative to the normal side, the lesion is most likely to be ligamentous.
○ If the rotation plus compression is more painful or shows decreased rotation relative to the normal side, the lesion is most likely to be a meniscus injur.

How well did you know this?
1
Not at all
2
3
4
5
Perfectly
61
Q

A 77 year-old female sustains a ground-level fall. She is brought to the ED where she complains of severe right groin pain. X-rays of her right hip reveal a complete fracture line across the femoral neck without displacement. What is the Garden classification of this fracture?

A Stage 4
B Stage 3
C Stage 2
D Stage 1

A

Answer: C

Explanation:
• This patient has suffered a Garden Stage 2 fracture.
• The Garden classification system of femoral neck fractures (intracapsular = within the hip capsule) is as follows.
• Stage 1: incomplete fracture line, nondisplaced.
• Stage 2: complete fracture line (fracture extends all the way across the femoral neck), nondisplaced.
• Stage 3: complete and partially displaced.
• Stage 4: complete and fully displaced (capsule is completely torn).

How well did you know this?
1
Not at all
2
3
4
5
Perfectly
62
Q

A 45 year-old male presents to your pain clinic with complaints of left elbow pain. On exam you note pain in the muscles and tendons just distal to the medial epicondyle. This patient’s condition is typically caused by which of the following?

A Excessive varus forces
B Excellent sporting technique
C Trauma
D Overuse

A

Answer: D

Explanation:
• Golfer elbow (medial epicondylitis) is inflammation of the common flexor tendon of the elbow due to microtears/overuse of the wrist and finger flexors, which originate at the common flexor tendon off the medial epicondyle.
• It is typically caused by overuse and poor technique.

How well did you know this?
1
Not at all
2
3
4
5
Perfectly
63
Q

A 32 year-old male suffers a fall onto his outstretched hand. He has immediate shoulder disfigurement which is corrected by a bystander. In the ED, MRI of the shoulder reveals an anterior labrum injury. This is commonly known by which of the following names?

A Bankart lesion
B Westfall lesion
C Hill-Sachs lesion
D O’Brien lesion

A

Answer: A

Explanation:
• This patient has suffered an anterior shoulder dislocation.
• MRI evidence of anterior labrum injury in the setting of shoulder disfigurement (= dislocation) is known as a Bankart lesion.
• Hill-Sachs lesion is a posterolateral humeral head compression fracture as a result of an anterior shoulder dislocation.
• The O’Brien and Westfall “lesions” are fictional.

How well did you know this?
1
Not at all
2
3
4
5
Perfectly
64
Q

A 24 year-old male, stiff-arming the competition in football, is about to score a touchdown when he decides to juke his defender. He plants his right foot on the ground and attempts to juke to the left when he experiences sudden-onset right knee pain and swelling. You rush onto the field to examine him. Anterior drawer and Lachman tests are negative. What is the most likely diagnosis?

A ACL tear
B PCL tear
C MCL tear
D Quadriceps tendon rupture

A

Answer: A

Explanation:
• An athlete attempting a noncontact cutting maneuver and experiencing sudden knee pain with instant swelling is a classic example of an ACL tear.
• It does not matter if Lachman/anterior drawer are negative; they could be falsely negative due to muscle spasm.
• The other structures are possibly torn in this patient, but the ACL should remain the highest on your differential in this scenario.

How well did you know this?
1
Not at all
2
3
4
5
Perfectly
65
Q

Which of the following is the most common cause of spinal cord injury?

A Violence
B Sports
C Motor vehicle accidents
D Falls

A

Answer: C

Explanation:
• Motor vehicle accidents remain the most common cause of SCI.
• Falls are the next most common cause.

How well did you know this?
1
Not at all
2
3
4
5
Perfectly
66
Q

A 14-year old male complains of bilateral knee pain of gradual onset for the past 2 months. He denies trauma or neurologic changes. His pain is located inferior to the patella, is worse during his basketball games, and is improved with ice. Knee x-rays demonstrate irregularities of the tibial tubercles. Which of the following is the most likely etiology of this patient’s pain?

A Ligament tear
B Quadriceps tendon inflammation
C Abnormal patellar tracking
D Traction apophysitis

A

Answer: D

Explanation:
• This patient presents with Osgood-Schlatter Disease (OSD).
• This is an overuse injury of the proximal tibia, usually due to excessive jumping activity involving the legs (e.g. basketball/volleyball) as the patellar tendon pulls on the tibia, leading to tibial tubercle traction apophysitis and fragmentation.
• X-rays may be normal or show tibial tubercle irregularities.
• Abnormal patellar tracking refers to patellofemoral pain syndrome, another common source of knee pain; however, this would likely NOT be solely located below the patella, and would NOT show tibial tubercle irregularities on X-ray.
• Quadriceps tendon inflammation would cause more superior pain near the quadriceps tendon, not inferior to the patella.
• Ligament tear would typically be associated with an acute presentation and a history of trauma, along with positive physical exam findings stressing the ligament in question and reproducing the patient’s pain.

How well did you know this?
1
Not at all
2
3
4
5
Perfectly
67
Q
  1. A 13 year-old male presents with back pain. He denies trauma. On exam you notice a forward-flexed posture. Spine x-rays demonstrate thoracic kyphosis of 50 degrees with anterior vertebral body wedging and Schmorl nodes in multiple segments. Which of the following is the most likely diagnosis?

A Friedreich ataxia
B Becker muscular dystrophy
C Scheuermann disease
D Idiopathic scoliosis

A

Answer: C

Explanation:
• This patient presents with Scheuermann disease, involving idiopathic juvenile kyphosis which can lead to a restrictive lung pattern.
• X-rays show kyphosis, vertebral body wedging, and Schmorl nodes.
○ Schmorl node: a common spinal disc herniation in which the soft tissue of the intervertebral disc bulges out into the adjacent vertebrae through an endplate defect.
• Treatment is physical therapy, bracing, and surgery.
• Scoliosis would show a coronal plane curvature, not simply kyphosis.
• Becker muscular dystrophy would also present with gradual onset weakness and disability.
• Friedreich ataxia would also demonstrate weakness, disability, cardiomyopathy, and vision and hearing dysfunction.

How well did you know this?
1
Not at all
2
3
4
5
Perfectly
68
Q

Which of the following muscles is the primary supinator of the forearm?

A Biceps brachii
B Supinator
C Brachialis
D Pronator teres

A

Answer: A

Explanation:
• Biceps brachii is the most powerful supinator of the forearm. This has been known to cause embarrassment for the muscle known as “supinator”.
• Biceps brachii: innervated by musculocutaneous nerve
• Supinator: innervated by posterior interosseous nerve (a branch of radial nerve)

How well did you know this?
1
Not at all
2
3
4
5
Perfectly
69
Q

Which of the following types of exercise places the greatest stress upon a tendon, leading to highest risk for tendon rupture?

A Fast eccentric
B Slow eccentric
C Fast concentric
D Slow concentric

A

Answer: A

Explanation:
• Concentric contractions occur with the muscle shortening as it contracts against a load.
• Eccentric contractions occur with the muscle lengthening as it fights to contract against this lengthening force.
• Fast eccentric contractions place the most acute stress upon a tendon, leading to highest risk of tendon rupture.

How well did you know this?
1
Not at all
2
3
4
5
Perfectly
70
Q

You are evaluating a patient for posterior heel pain. A lower extremity MRI reveals a complete tear of the achilles tendon. However, when you perform the Thompson test, it is negative. How is this possible?

A It is simply a false positive Thompson test
B A bifid achilles tendon is present
C The MRI diagnosis is incorrect
D The plantaris is intact

A

Answer: D

Explanation:
• In a patient with complete achilles tendon rupture, the Thompson test should be positive.
• Recall that a positive Thompson test is detected by squeezing the patient’s calf and documenting a lack of plantarflexion, indicating a complete achilles tendon tear.
• If a patient has a confirmed achilles tendon rupture on MRI, yet demonstrates a falsely negative Thompson test, this seems like it would be impossible, but in reality the examiner is likely squeezing enough to cause contraction of the plantaris tendon, which also attaches to the calcaneus, and can perform plantarflexion.
• In summary, squeezing the calf does NOT only squeeze the gastrocnemius and soleus, but also the plantaris muscle and tendon, which sit between the gastrocnemius and soleus, and all of these muscles may provide plantarflexion function to the ankle.
• An MRI-confirmed achilles tendon rupture is NOT likely to be incorrect.
• Naturally occurring bifid achilles tendons are fictional.
• This Thompson test is a false negative result, not a false positive.

How well did you know this?
1
Not at all
2
3
4
5
Perfectly
71
Q

With knee extension the ACL (anterior cruciate ligament of the knee) becomes which of the following?

A Loose
B Tight
C Neither
D Both

A

Answer: B

Explanation:
• The ACL runs anteroinferomedially to insert onto the tibia. It tenses with knee extension and limits anterior translation of the tibia (hence the Anterior Drawer and Lachman tests.
• With knee flexion the ACL pulls the femur anteriorly.

How well did you know this?
1
Not at all
2
3
4
5
Perfectly
72
Q

You are performing a physical exam on a patient with knee pain. You flex the hip and knee, internally rotate the tibia, and apply a varus force to the knee while extending the knee. This test is known typically by what name?

A Apley
B Thessaly
C McMurray
D Bounce Home

A

Answer: C

Explanation:
• The McMurray test is a test for meniscus tears in the knee.
• By externally rotating the ankle and applying a valgus force while extending the knee, you are testing the medial meniscus for tears.
• Internal rotation the ankle with varus force is a test for the lateral meniscus.
• Both are called the McMurray test.
• Tip: the heel itself moves toward the meniscus being tested; i.e. with external rotation, the heel moves medially, and thus, the medial meniscus is tested (with external rotation + valgus stress to the knee).

How well did you know this?
1
Not at all
2
3
4
5
Perfectly
73
Q

Rheumatoid factor is found in rheumatoid arthritis and which of the following other diseases?

A Psoriatic arthritis
B CREST syndrome
C Sjogren syndrome
D Systemic lupus erythematosus (SLE)

A

Answer: C

Explanation:
• Rheumatoid factor (RF) can be found in rheumatoid arthritis patients as well as those with Sjogren syndrome.

How well did you know this?
1
Not at all
2
3
4
5
Perfectly
74
Q

The scarf test is primarily a test for which joint?

A Cervical spine
B Scapulothoracic
C Glenohumeral (GHJ)
D Acromioclavicular (AC)

A

Answer: D

Explanation:
• Scarf test is performed by adducting the affected shoulder across midline, grabbing the other shoulder in the process and inducing pain if the patient has disease within the AC joint.
○ The test is performed by passively bringing the patient’s arm into 90 degrees of forward flexion, with their elbow also flexed to 90 degrees. The examiner then horizontally adducts the flexed arm across the patient’s body, bringing their elbow towards the contralateral shoulder.
• Thus, this is a test for the AC joint.
• Adducting the right shoulder across midline is the scarf test for the right AC joint.

How well did you know this?
1
Not at all
2
3
4
5
Perfectly
75
Q

A 31 year-old female presents to your clinic with 6 weeks of anterior left ankle pain. She denies trauma, weakness, numbness, or tingling. She is a soccer player and has been playing lots of tournaments lately. On exam, passive plantarflexion reproduces her pain, as does resisted dorsiflexion. Interestingly she also has pain with resisted inversion. Ankle x-rays are normal. What is the next best step?

A Corticosteroid injection
B Orthopedic surgery consultation
C Rest, ice, NSAIDs
D Ankle MRI

A

Answer: C

Explanation:
• This patient’s symptoms are suspicious for left tibialis anterior tendonopathy/tenosynovitis.
• Initial treatment involves rest, ice, NSAIDs, and gradual return to increasing levels of activity as tolerated. Physical therapy is also beneficial. Corticosteroid injection into the tendon sheath can help in resistant cases.
• MRI is not needed for this diagnosis.
• Surgery is indicated in cases of tendon rupture.

How well did you know this?
1
Not at all
2
3
4
5
Perfectly
76
Q

A 34 year-old male is snowboarding down some intense moguls. The moguls get the best of him, and he is launched into the air. He lands awkwardly and sustains a left tibial fracture. Snow patrol is called. The patient is initially resting on the ground in tolerable pain, but as the minutes pass he increasingly moans in pain. Snow patrol arrives, and passive plantarflexion causes the patient to scream in extreme pain. What is probably going to be the most important early intervention for this patient?

A Emergent fasciotomy
B Emergent casting
C Emergent pain control
D Emergent fracture repair

A

Answer: A

Explanation:
• Acute compartment syndrome is an emergency condition in which the pressure within a muscle compartment (usually the lower leg or the forearm) rises so quickly, and to such high levels, such that blood can pump in, but it cannot pump out; this is usually in the setting of trauma/fracture, and most commonly takes place in the anterior compartment.
• Passive stretch of the muscle within that compartment causes extreme pain out of proportion to the injury.
• If allowed to progress, the patient will develop pain, paresthesias due to nerve compression, and paralysis due to nerve compression and ischemia.
• Workup involves compartment pressure testing via needle manometry.
• Treatment is emergent fasciotomy to avoid longterm sequelae such as sensory abnormalities and permanent weakness.

How well did you know this?
1
Not at all
2
3
4
5
Perfectly
77
Q

All of the following bones belong in the distal row of carpal bones except which of the following?

A Hamate
B Trapezoid
C Triquetrum
D Trapezium

A

Answer: C

Explanation:
• The proximal row of carpal bones contains scaphoid, lunate, triquetrum, and pisiform.
• The distal row of carpal bones contains trapezium, trapezoid, capitate, hamate.
• “Some Lovers Try Positions That They Can’t Handle.”

How well did you know this?
1
Not at all
2
3
4
5
Perfectly
78
Q

A 63 year-old female presents to your PM&R clinic with complaints of left wrist swelling. She denies pain, numbness, tingling, or weakness. She says she has had this swelling for several months, and has noticed fluctuations in its size. On exam, it is not tender to palpation, and it is soft and compressible. Her skin color over the swelling appears normal. What is the most likely underlying pathophysiology for this process?

A Tendon swelling due to chronic disease
B Outpouching of synovial fluid
C Infection
D Benign adipose cell proliferation

A

Answer: B

Explanation:
• This patient presents with findings suspicious for ganglion cyst, which is typically a painless outpouching of synovial fluid from a joint space or tendon sheath.
• Management may include observation, aspiration, or surgical removal.

How well did you know this?
1
Not at all
2
3
4
5
Perfectly
79
Q

The “roof” of the tarsal tunnel is also known as which of the following?

A Ligament of Johnson
B Flexor retinaculum
C The tarsal conduit
D Extensor retinaculum

A

Answer: B

Explanation:
• The roof of the tarsal tunnel can be thought of as the flexor retinaculum, under which the structures of the tarsal tunnel pass.
• Tarsal tunnel syndrome is a compression, or squeezing, on the posterior tibial nerve.
• Tarsal tunnel contains Tibialis posterior tendon, Flexor digitorum longus tendon, Tibial vein, Tibial artery, Tibial nerve, Flexor hallucis longus tendon.

How well did you know this?
1
Not at all
2
3
4
5
Perfectly
80
Q

The ACL’s (anterior cruciate ligament of the knee) primary function is to do which of the following?

A Prevent posterior tibial translation
B Prevent posterior femur translation
C Prevent anterior tibial translation
D Prevent posterior femur translation

A

Answer: C

Explanation:
• The ACL runs anteroinferomedially to insert onto the tibia. It tenses with knee extension and limits anterior translation of the tibia (hence the Anterior Drawer and Lachman tests. With knee flexion the ACL pulls the femur anteriorly.

How well did you know this?
1
Not at all
2
3
4
5
Perfectly
81
Q

You are performing a physical exam on a patient with knee pain. You flex the hip and knee, externally rotate the ankle, and apply a valgus force to the knee while extending the knee. Which structure are you assessing for a tear?

A Medial meniscus
B Lateral meniscus
C MCL
D LCL

Answer: A

Explanation:
• McMurray tes is a test for meniscus tears in the knee.
• By externally rotating the ankle and applying a valgus force while extending the knee, you are testing the medial meniscus for tears; internal rotation with varus force is a test for the lateral meniscus. Both are called the McMurray test.

A
  1. You are performing a physical exam on a patient with knee pain. You flex the hip and knee, externally rotate the ankle, and apply a valgus force to the knee while extending the knee. Which structure are you assessing for a tear?

A Medial meniscus
B Lateral meniscus
C MCL
D LCL

Answer: A

Explanation:
• McMurray tes is a test for meniscus tears in the knee.
• By externally rotating the ankle and applying a valgus force while extending the knee, you are testing the medial meniscus for tears; internal rotation with varus force is a test for the lateral meniscus. Both are called the McMurray test.

How well did you know this?
1
Not at all
2
3
4
5
Perfectly
82
Q

Which of the following is a known sequela of rheumatoid arthritis?

A Ulnar deviation of the wrist; ulnar deviation of the fingers
B Ulnar deviation of the wrist; radial deviation of the fingers
C Radial deviation of the wrist; radial deviation of the fingers
D Radial deviation of the wrist; ulnar deviation of the fingers

A

Answer: D

Explanation:
• In rheumatoid arthritis, one of the sequelae is a deformity resulting in radial wrist deviation with ulnar finger deviation.

How well did you know this?
1
Not at all
2
3
4
5
Perfectly
83
Q

A 75 year-old male with history of bilateral knee replacements presents to your clinic with complaints of right groin pain. He denies trauma. It has developed gradually over the past several months along with low back pain. He denies numbness or tingling, but feels weak in his right leg. On exam, strength is neurologically intact. There is no tenderness to palpation of the anterior, lateral, or posterior right hip. FABERE reproduces his right groin pain, but not back pain. What is the most likely diagnosis?

A Iliopsoas tendonitis
B Hip osteoarthritis
C Lumbar radiculitis
D Labral tear of hip

A

Answer: B

Explanation:
• This patient presents with classic gradual onset osteoarthritis of the hip. True hip pain manifests as groin pain.
• An aging individual with a history of presumably knee OA leading to total knee replacements and gradual onset groin pain without trauma is most likely indicating “wear and tear” femoroacetabular joint arthritis.
• There is NO reason to suspect a labral tear in this patient without trauma and presenting to you for the first time.
• Further, an elderly patient presenting in this fashion would suggest osteoarthritis before you should consider a labral tear.
• Tendons are nontender on exam, and there is low suspicion for radicular pathology, given that his exact groin pain was reproduced on exam with FABERE testing.

How well did you know this?
1
Not at all
2
3
4
5
Perfectly
84
Q

A 37 year-old male presents with 7 months of gradual onset low back pain. He denies trauma. On exam while supine, hanging one leg off the edge of the exam table reproduces his low back pain. What is the name of this maneuver?

A Gillet
B FABER
C Yeoman
D Gaenslen

A

Answer: D

Explanation:
• This question describes the Gaenslen test.
• A positive test reproducing low back pain is suggestive of sacroiliac joint dysfunction/pain.

How well did you know this?
1
Not at all
2
3
4
5
Perfectly
85
Q

You are the sideline physician at a basketball game when a 23 year-old male falls to the floor, landing on his right shoulder. There is visible skin tenting superficial to the distal end of the clavicle. He is in significant pain. What is the underlying pathology causing his presentation?

A Tear of both the acromioclavicular and coracoclavicular ligaments
B Tear of the coracoclavicular ligament
C Tear of the acromioclavicular ligament
D Tear of the superior glenohumeral ligament

A

Answer: A

Explanation:
• This patient has sustained an acromioclavicular (AC) joint separation. The AC and CC ligaments are the ligaments implicated in this process.
• The AC ligament is typically first to tear, followed by the CC ligament.
• When they are both torn, the clavicle is no longer anchored down and becomes displaced, thus floating upward and causing the skin tenting seen here.

How well did you know this?
1
Not at all
2
3
4
5
Perfectly
86
Q

A 50 year-old female presents with bilateral knee pain of gradual onset for the past 5 months. She says she first noticed the pain after a long day of moving furniture. She has felt fatigued with occasional fever. She also notes 60-90 minutes of knee stiffness when she wakes up. Knee x-rays reveal symmetric medial and lateral compartment narrowing. Which of the following is the most likely diagnosis?

A Septic arthritis
B Lupus arthritis
C Rheumatoid arthritis
D Osteoarthritis

A

Answer: C

Explanation:
• Rheumatoid arthritis presents with several weeks at least of joint pain involving typically the knees, MCPs, PIPs, MTPs, C1-C2 joint) along with > 1 hour of morning stiffness in addition to fatigue and fever.
• X-rays will show symmetric joint space narrowing, rather than asymmetric narrowing typical of osteoarthritis.
• Septic arthritis is more likely to be monoarticular with persistent fever and joint swelling.
• Lupus arthritis would also present with other symptoms of lupus, such as malar rash, photosensitivity, mouth ulcers, pericarditis, pleuritis, etc.

How well did you know this?
1
Not at all
2
3
4
5
Perfectly
87
Q

A patient with a complete tear of the acromioclavicular (AC) ligament, complete tear of the coracoclavicular (CC) ligament, and clavicle that is displaced inferiorly has what grade of AC joint separation, and what should the treatment be?

A 7, surgery
B 6, surgery
C 5, surgery
D 4, surgery

A

Answer: B

Explanation:
• AC joint separations occur as grades 1-6.
• Grade 1: partial AC tear, intact CC, rehab.
• Grade 2: complete AC tear, partial CC tear, rehab.
• Grade 3: complete AC and CC tears, clavicle floats upward, rehab vs. surgery.
• Grade 4: complete AC and CC tears, clavicle floats superiorly and posteriorly, surgery.
• Grade 5: complete AC and CC tears, clavicle floats even more superiorly and posteriorly, surgery.
• Grade 6: complete AC and CC tears, clavicle floats downward, surgery.

How well did you know this?
1
Not at all
2
3
4
5
Perfectly
88
Q

The conjoint hamstring tendon houses which tendons?

A Semimembranosus, semitendinosus
B Semimembranosus, biceps femoris
C Semitendinosus, biceps femoris
D None of the above combinations

A

Answer: C

Explanation:
• The conjoint hamstring tendon contains semitendinosus and biceps femoris tendons medially on the ischial tuberosity.
• The semimembranosus tendon originates laterally from the other two tendons (paradoxically, as it becomes the most medial hamstring muscle as the muscles extend distally).

How well did you know this?
1
Not at all
2
3
4
5
Perfectly
89
Q

Typically the 2nd ligament to be torn in a lateral ankle sprain is which of the following? ATFL: anterior talofibular ligament. CFL: calcaneofibular ligament. PTFL: posterior talofibular ligament.

A Deltoid ligament
B PTFL
C ATFL
D CFL

A

Answer: D

Explanation:
• Generally the order of ligaments most commonly torn in a lateral ankle sprain is:
• Anterior talofibular ligament > Calcaneofibular ligament > Posterior talofibular ligaemnt.
• Deltoid ligament is torn in medial ankle sprains.

How well did you know this?
1
Not at all
2
3
4
5
Perfectly
90
Q

A 78 year-old male with a history of poorly controlled diabetes mellitus presents to your clinic with 1 year of progressive right knee pain. He denies trauma history. Acetaminophen provides some relief. He notes numbness and tingling in his toes bilaterally. He notes low back pain with this. He denies bowel or bladder dysfunction. On exam he has intact strength, sensation, and reflexes. He has tenderness to palpation of his right knee medial joint line. McMurray test is positive. He ambulates with a Trendelenburg lean to the right. What is the most appropriate next step?

A Knee x-rays
B Physical therapy
C Knee MRI
D Corticosteroid injection

A

Answer: B

Explanation:
• This patient demonstrates classic symptoms of knee osteoarthritis (OA).
• He has numbness and tingling from diabetes as a red herring here. He also has back pain, most likely as a result of his gait dysfunction due to the osteoarthritis.
• The first step in treating knee OA is to initiate physical therapy.
• If he fails physical therapy, knee X-rays should be considered to evaluate the degree of the suspected OA, and a corticosteroid injection into the knee could be considered vs. other regenerative injection options for the knee.
• MRI of the knee is useful for identifying meniscal tears, which this patient indeed may have, but MRI of the knee would not be the first step here, as minor meniscal tears should be rehabilitated first instead of surgically intervened upon.

How well did you know this?
1
Not at all
2
3
4
5
Perfectly
91
Q

A 72 year-old male presents to your clinic with complaints of 1 year of progressive left thumb pain. He has tried acetaminophen, NSAIDs, and a thumb spica splint to minimal relief. He denies weakness, numbness, or tingling, but says it has become more difficult to open doors and perform household tasks due to the pain. On exam, applying an axial force to the thumb while rotating it around reproduces the patient’s pain. What is the next best step?

A Surgical referral
B Corticosteroid injection
C EMG
D MRI

A

Answer: B

Explanation:
• This patient presents with first CMC arthritis, which is typically a “wear and tear” condition of the base of the thumb.
• 1st CMC grind test is positive, as described in the question, indicated 1st CMC arthritis.
• Conservative treatments of splinting and oral medications have failed.
• A reasonable next step would be to trial a corticosteroid injection to provide relief for him.
• Surgery would be a last resort.
• EMG is not indicated in a clear clinical diagnosis of 1st CMC OA without any numbness, tingling, or weakness.
• X-ray would be appropriate as an initial imaging test, but not MRI.

How well did you know this?
1
Not at all
2
3
4
5
Perfectly
92
Q

A 67 year-old female with a T score of -2.7 on a most recent DXA scan asks you for advice regarding bone health. Which of the following is the most reasonable action to take to improve her bone health?

A Limit how often she walks
B Canoeing
C Swimming
D Resistance exercises

A

Answer: D

Explanation:
• This patient with T score -2.7 fits DXA criteria for osteoporosis.
• In order to improve her bone density, weight-bearing exercise is recommended, such as resistance training, ambulation, stair-climbing, golf, tennis, etc.
• Swimming and canoeing are excellent physical activities, but will not offer maximal impact in terms of improving her bone health.
• Putting limits on this patient’s walking will only encourage disability.

How well did you know this?
1
Not at all
2
3
4
5
Perfectly
93
Q

A 26 year-old male falls on his outstretched hand and sustains a right humeral midshaft fracture. If you were to find weakness in this patient’s right upper limb, which muscle would be most likely to be weak following this fracture?

A Flexor carpi radialis
B Extensor indicis proprius
C Triceps
D Flexor carpi ulnaris

A

Answer: B

Explanation:
• Midshaft humeral fractures raise suspicion for radial nerve injury in the spiral groove.
• Triceps and Anconeus are already innervated at this point, so we must look for any distal radial nerve-innervated muscle.
• Extensor indicis proprius is the only muscle that fits this category, as it has posterior interosseous nerve innervation which itself is a branch of the radial nerve.

How well did you know this?
1
Not at all
2
3
4
5
Perfectly
94
Q

When extended, the human elbow typically demonstrates what type of natural alignment?

A Curved
B Varus
C Valgus
D Straight

A

Answer: C

Explanation:
• There is slight normal valgus angulation of the elbow when extended.

How well did you know this?
1
Not at all
2
3
4
5
Perfectly
95
Q

A patient develops gradual onset knee pain due to a redundant fold of synovial tissue in the knee that has become thickened and inflamed, leading to knee pain with locking and catching of the knee. The patient has tried physical therapy and corticosteroid injection to only minimal relief. They ask what else they should consider. What is your response?

A Surgery
B Knee bracing
C Prolotherapy injection
D Pain management

A

Answer: A

Explanation:
• This question describes a knee plica causing pain and inflammation.
• It is best treated with physical therapy, corticosteroid injection into the plica, or surgical resection of the plica.
• Synovial plica syndrome: Patients often mention anterior knee pain, clicking, clunking, and a popping sensation on patellofemoral loading activity such as squatting.
○ According to their location, the synovial plicae are classified as suprapatellar, mediopatellar, infrapatellar, or lateral; the medial plica is the most commonly symptomatic one.
○ A pathological synovial plica that has been through this inflammatory process can become inelastic, tight, thickened, fibrotic, and sometimes hyalinized.
○ A synovial plica affected by such changes may bowstring across the femoral trochlea, causing impingement between the patella and femur in knee flexion.

How well did you know this?
1
Not at all
2
3
4
5
Perfectly
96
Q

The lateral collateral ligament (LCL) of the knee originates on the lateral femoral condyle and inserts onto which of the following structures?

A Fibular head
B Gerdy’s tubercle of the tibia
C Pes anserine of the tibia
D Fibular shaft

A

Answer: A

Explanation:
• The LCL originates on the lateral femoral condyle and attaches onto the fibular head.

How well did you know this?
1
Not at all
2
3
4
5
Perfectly
97
Q

A 21-year-old basketball player attempts to catch a pass from his teammate but drops the ball and immediately experiences pain in his distal third digit. Instant replay shows the basketball hitting his third fingertip causing a forced flexion moment at the DIP. He is taken out of the game and seen by the team physician; during initial examination, the patient is unable to actively extend the DIP of his third finger. Xrays are negative for acute fracture. What splint is most appropriate to promote proper healing of his injury?

A Stax splint
B Boutonniere ring splint
C Swan neck ring splint
D Resting hand splint

A

Answer: A

Explanation:
• The vignette describes a mallet finger injury, which is often seen in sports such as basketball or baseball.
○ Typically, a ball hits the distal aspect of a finger, causing flexion of the DIP and resultant extensor tendon rupture. This leads to inability to actively extend the DIP.
○ X-rays are appropriate to rule out avulsion fracture.
○ A stax splint or DIP extension splint is appropriate to allow for healing of the DIP extensor tendon.
• A swan-neck ring splint and Boutonniere ring splint are examples of static-motion blocking splints that are used to treat their respective namesake.
• A resting hand splint promotes ROM of joints of the hand to avoid contracture formation, typically after stroke.

How well did you know this?
1
Not at all
2
3
4
5
Perfectly
98
Q

A patient develops sudden onset calf pain when playing basketball. On exam, you squeeze his calf muscle and watch as his ankle plantarflexes during the squeeze. What is the name of this test?

A Johannson test
B Thompson test
C Thomas test
D Johnson test

A

Answer: B

Explanation:
• This question describes the Thompson test.
• In a positive test, the ankle will fail to plantarflex during the calf squeeze, indicating a ruptured achilles tendon.

How well did you know this?
1
Not at all
2
3
4
5
Perfectly
99
Q

The function of the anterior talofibular ligament (ATFL) is to do which of the following?

A Resist lateral translation of the talus
B Stabilize the medial ankle by resisting eversion forces
C Resist anterior translation of the talus
D Stabilize the lateral ankle by resisting eversion forces

A

Answer: C

Explanation:
• The function of the anterior talofibular ligament (ATFL) is to resist anterior talar translation.
• The ATFL’s integrity can be tested using the anterior drawer test of the ankle, in which the examiner stabilizes the calf distally and pulls anteriorly on the calcaneus, thus drawing the talus anteriorly, separating it away from the fibula and, thus, stretching/stressing the ATFL.
• If it is torn, there will be NO firm endpoint.

How well did you know this?
1
Not at all
2
3
4
5
Perfectly
100
Q

A 34 year-old male falls off a ladder and lands on his feet. He soon develops right midfoot pain. You suspect a right Nutcracker fracture. In addition to foot x-rays to confirm the diagnosis (because your diagnostic skills are supreme), what will be your recommendation?

A Soft custom insole
B Orthopedic surgery consult
C MRI
D EMG

A

Answer: B

Explanation:
• Nutcracker fractures (cuboid fractures) require X-rays and orthopedic surgery consultation.

How well did you know this?
1
Not at all
2
3
4
5
Perfectly
101
Q

A 32 year-old female presents to you with 3 weeks of right lateral elbow pain. She has been working to improve her tennis game lately and has been taking lessons daily. She is a big fan of Roger Federer. On exam, you elicit concordant elbow pain with resisted wrist extension. Which of the following is an appropriate recommendation for this patient?

A Increase string tension
B Decrease string tension
C Play on grass courts
D Play on hard courts

A

Answer: B

Explanation:
• With tennis elbow (lateral epicondylitis, described in the question stem), it is important to help repair the common extensor tendon at the elbow (typically the extensor carpi radialis brevis - ECRB - bears most of the pain in tennis elbow).
• We do this by following RICE (rest, ice, compression, elevation).
• Once acute inflammation resolves and the patient is pain-free at rest, physical therapy to strengthen and stretch the common extensor tendon is important.
• Other important ways to reduce sudden inappropriate forces through the common extensor tendon include decreasing string tension on the racquet, playing on slower courts (e.g. clay), increasing grip size, and improving swing technique, notably the backhand technique.

How well did you know this?
1
Not at all
2
3
4
5
Perfectly
102
Q

A patient with a complete tear of the acromioclavicular (AC) ligament, complete tear of the coracoclavicular (CC) ligament, and clavicle that is displaced superiorly and posteriorly has what grade of AC joint separation, and what should the treatment be?

A 4, surgery
B 4, rehab
C 3, surgery
D 3, rehab

A

Answer: A

Explanation:
• AC joint separations occur as grades 1-6.
• Grade 1: partial AC tear, intact CC, rehab.
• Grade 2: complete AC tear, partial CC tear, rehab.
• Grade 3: complete AC and CC tears, clavicle floats upward, rehab vs. surgery.
• Grade 4: complete AC and CC tears, clavicle floats superiorly and posteriorly, surgery.
• Grade 5: complete AC and CC tears, clavicle floats even more superiorly and posteriorly, surgery.
• Grade 6: complete AC and CC tears, clavicle floats downward, surgery.

103
Q

A 60 year-old male with a history of hypertension and diabetes mellitus presents to your clinic with the chief complaint of left shoulder pain. He says it hurts “all the time”, and he hears popping and clicking sounds throughout the day. On exam, drop arm test is negative. There is crepitus upon passive range of motion. He experiences pain throughout the entire range of motion. Palpation around the glenohumeral joint causes pain. His strength is intact throughout the rotator cuff. What is the best initial action?

A Corticosteroid injection
B Physical therapy
C Shoulder MRI
D Shoulder X-rays

A

Answer: B

Explanation:
• This patient presents with likely glenohumeral joint osteoarthritis (OA).
• The first step in treating this patient would be to enroll in physical therapy to improve pain and function in the shoulder.
• If this fails, the next step would be to consider shoulder X-rays to evaluate the joint space.
• Shoulder MRI is useful if the physician suspects a soft tissue injury such as a rotator cuff tear.
• Corticosteroid injection into the shoulder would not be appropriate unless the patient fails conservative measures.

104
Q

A 58 year-old male presents with sudden-onset right knee pain and swelling. He denies trauma. Synovial fluid analysis reveals crystals with a positive birefringence pattern. What is the pathophysiology of this disease?

A Calcium pyrophosphate deposition
B Elevated acute phase reactants
C Abnormal red blood cell morphology
D Elevated uric acid

A

Answer: A

Explanation:
• Pseudogout causes sudden onset pain (usually in the knee) due to calcium pyrophosphate crystal deposition (also called chondrocalcinosis).
• Synovial fluid analysis reveals crystals with a positive birefringence pattern.
• Treatment is indomethacin, colchicine, and corticosteroids for acute attacks.

105
Q

A 46 year-old male with a history of hypertension, diabetes, and hyperlipidemia develops sudden-onset severe foot pain. He denies trauma. On exam you notice swelling of the great toe. Synovial fluid analysis reveals crystal deposition with a negative birefringence pattern. What is the most likely diagnosis?

A Gout
B Pseudogout
C Insufficiency fracture
D Septic arthritis

A

Answer: A

Explanation:
• Gout presents with sudden onset severe pain typically in the great toe with swelling in the joint. Synovial fluid analysis reveals crystals showing a negative birefringence pattern.
• Pseudogout crystals show a positive birefringence pattern (due to calcium pyrophosphate deposition).
• Fracture would generally have some physical activity or trauma history component. Septic arthritis would generally present with fever as well.

106
Q

A 21 year-old female complains of gradual onset worsening right hip pain. She is currently training for a marathon. She denies weakness, numbness, and tingling. X-rays reveal a cortical lucency along the inferior/medial femoral neck. There is pain during normal ambulation. What is the most appropriate next step in management?

A Physical therapy
B Make right leg non-weight-bearing
C Orthopedic surgery consult
D Closed reduction and casting

A

Answer: B

Explanation:
• This patient has developed a compression-side proximal femur stress factor.
• Risk factors include:
○ Low BMI
○ Poor nutrition
○ Amenorrhea
○ Osteoporosis
• Compression-side fractures (on inferior/medial side of femoral neck) are more common than tension-side fractures (fractures along superior/lateral femoral neck) and can be treated with non-weight-bearing for a period of time, then gradual weight-bearing as tolerated, then gradually increase activity.
• Tension-side fractures are less stable and require orthopedic surgery consult for ORIF.

107
Q

A Baker cyst is an inflammatory fluid collection of a bursa at the intersection of which of the following structures?

A Lateral head of gastrocnemius and semitendinosus
B Medial head of gastrocnemius and semitendinosus
C Lateral head of gastrocnemius and biceps femoris
D Medial head of gastrocnemius and semimembranosus

A

Answer: D

Explanation:
• A Baker cyst lies in the intersection between medial head of gastrocnemius and semimembranosus.

108
Q

The fourth extensor compartment of the wrist contains which of the following tendons?

A Extensor carpi radialis longus and brevis
B Extensor digitorum, extensor indicis proprius
C Extensor carpi ulnaris
D Extensor pollicis longus

A

Answer: B

Explanation:
• The fourth extensor compartment of the wrist contains the Extensor digitorum and Extensor indicis proprius tendons.
• 1st extensor compartment contains Abductor pollicis longus and Extensor pollicis brevis tendons.
• 2nd extensor compartment contains Extensor carpi radialis longus and Extensor carpi radialis brevis tendons.
• 3rd extensor compartment contains Extensor pollicis longus tendon.
• 4th extensor compartment contains Extensor digitorum and Extensor indicis.
• 5th extensor compartment contains the Extensor digiti minimi tendon.

109
Q

An 18-year old female ballet dancer complains of gradual onset right ankle and foot pain. Her pain is worse during dancing, and improved with rest. On exam, passive great toe extension reproduces her pain. Palpation of the proximal medial plantar surface just distal to the calcaneus does not cause pain. Palpation of the space just posterior to the medial malleolus causes pain. Ankle x-rays are negative for bony abnormality. What is the most likely mechanism of her problem?

A Injury/dysfunction of the flexor hallucis longus
B Excessive periosteal traction
C Overuse of the tibialis posterior
D Overuse of the extensor hallucis longus

A

Answer: A

Explanation:
• Ballet dancers are at risk for flexor hallucis longus tendonitis/tendonopathy/injury. This is generally an overuse injury, as some dancing disciplines require repeated great toe flexion against resistance.
• Treatment is generally rest, ice, NSAIDs, and physical therapy.
• Surgery if tendon rupture takes place.
• Recall that the flexor hallucis longus wraps around the tarsal tunnel (hence the pain when palpating there) and attaches to the great toe (hence the pain with stretching of the tendon by extending the great toe). Pain is also reproduced with resisted great toe flexion.
• X-rays have ruled out tibial stress fracture.
• Shin splints would generally cause tibial shin pain, not ankle and foot pain.

110
Q

A patient with sickle cell disease who has experienced chronic episodic left hip/groin pain will most likely suffer from which of the following pathologies later in his life?

A Femoroacetabular impingement
B Greater trochanteric bursitis
C Hip avascular necrosis (AVN)
D Hip labral tear

A

Answer: C

Explanation:
• AVN is a common sequela of chronic sickle cell disease, notably in the hip joint.

111
Q

A 47 year-old female presents to your office with complaints of 8 weeks of right buttock pain shooting down her right posterior thigh, stopping at the knee. She notes numbness and tingling in this pattern. She denies weakness or bowel/bladder dysfunction. She has tried PT but found it too painful to endure. Ibuprofen has provided minimal relief. On exam, strength, sensation, and reflexes are normal. She is exquisitely tender to palpation of the lateral right buttock, reproducing her buttock pain. Slump sit and FAIR tests are positive on the right. MRI of the lumbar spine is unremarkable. What is the next best step?

A EMG right lower limb
B EMG bilateral lower limbs
C US-guided right piriformis injection
D Reassurance and encourage PT

A

Answer: C

Explanation:
• This patient presents with classic piriformis syndrome.
• Buttock pain is reproduced with piriformis palpation and FAIR maneuver, with some sciatic nerve irritation symptoms (numbness/tingling and radiating pain down the posterior/lateral thigh).
○ FAIR stands for flexion, adduction and internal rotation. Also known as piriformis test.
• US-guided piriformis injection is typically well tolerated and can be a quick, in-office solution to piriformis pain, and should be followed by resumption of physical therapy.
• In cases of clear piriformis syndrome, such as this case, piriformis injection is useful.
• In less clear cases, one may opt for a lumbar epidural steroid injection, such as at L5 vs. S1.
• EMG would NOT be useful in this patient without functional disturbance or clinical weakness, whose pain is reproducible with palpation of the offending muscle.

112
Q

A 29 year-old male long distance runner presents to you with gradual onset bilateral anterior knee pain. His pain is worse when descending stairs. He has occasional knee stiffness after prolonged sitting. On exam, he has a positive patellar grind test. Knee x-rays reveal a shallow medial patellofemoral contour compared to typical normal studies. What is the next best step?

A IT band stretching and vastus lateralis strengthening
B IT band strengthening and vastus medialis stretching
C Vastus medialis strengthening and vastus lateralis stretching
D Vastus lateralis strengthening and vastus medialis stretching

A

Answer: D

Explanation:
• This patient presents with classic patellofemoral pain syndrome (PFPS).
• This is abnormal patellar tracking due to muscle weakness and imbalances; specifically this typically involves vastus lateralis and IT band tightness in combination with vastus medialis weakness.
• Treatment typically involves vastus medialis strengthening with vastus lateralis and IT band stretching, along with hip girdle strengthening for stability.
• Sometimes patellar knee sleeves are used to assist with the proprioception of proper patellar tracking, as is kinesiotaping.
• The patellar grind test is noted here, which is a test for PFPS.
• If rest, physical therapy, NSAIDs, and bracing do not improve the pain, MRI of the knee and surgery may be considered.
• Note: typically this pattern described here is the muscular pattern of weakness and tightness for PFPS, but it is possible to have a weak vastus lateralis and tight vastus medialis and perform essentially the opposite PT as for typical PFPS.
• This “opposite pattern” includes vastus medialis stretching and vastus lateralis strengthening to try to realign the patella to track properly.
• That is the correct answer per this patient’s history and examination in combination with X-ray studies.
• X-rays may be taken to help diagnose PFPS, especially in cases refractory to generic PT; X-rays may show abnormal patellofemoral contours which may provide a clue as to which direction the patella is tracking into (improperly tracking, to be precise).

A shallower-than-normal patellofemoral contour would imply that the patella does NOT “stay in its lane”, and may easily veer off towards the shallow side, since it does NOT have good bony anatomy keeping it in place. This veering off is the reason for these patients’ knee pain.

113
Q

A 30 year-old male develops left knee pain and swelling associated with fever. Synovial fluid analysis shows 130k WBCs with 97% neutrophils. What is the most likely etiology for this disease?

A Staph aureus
B Calcium pyrophosphate deposition
C Uric acid deposition
D Neisseria gonorrhea

A

Answer: D

Explanation:
• Septic arthritis in adults is typically due to N. gonorrhea infection.
• Synovial fluid analysis in septic arthritis reveals > 100k WBCs and overwhelming neutrophil (PMN) predominance.
• Septic arthritis in children is typically due to staph aureus.
• Crystal arthropathy (gout, pseudogout) would not show these synovial fluid findings (there would be < 100k WBCs and around 75% neutrophils).

114
Q

The “OK” sign is a test for which of the following nerves?

A Posterior interosseous nerve
B Radial nerve
C Anterior interosseous nerve
D Median nerve

A

Answer: C

Explanation:
• The “OK” sign is performed by asking the patient to flex their thumb interphalangeal joint and flex the distal interphalangeal (DIP) joint of the index finger while extending digits 3-5 to make the classic “OK” sign.
• In patients with anterior interosseous neuropathy (AIN-opathy), thumb flexion (flexor pollicis longus) and DIP flexion (flexor digitorum profundus) will be affected, as these are both AIN functions.
• These patients will essentially keep a straight finger and thumb while pinching them together, instead of flexing these joints properly
• Note: the Froment sign (ulnar neuropathy) and the “OK” sign (AIN-opathy) are essentially opposite tests of each other!
○ Froment’s sign presents after damage to the ulnar nerve, which innervates the adductor pollicis and interossei muscles, which provide adduction of the thumb and extension of the interphalangeal joint.
○ The flexor pollicis longus (innervated by the median nerve), will substitute for the adductor pollicis (innervated by the ulnar nerve) and cause the thumb to go into hyperflexion.

115
Q

An 80 year-old female presents to you with 6 months of right foot pain with stiffness. On exam, you localize the pain and stiffness to the great toe. Strength and sensation are intact. X-rays demonstrate joint space narrowing with osteophytosis. In addition to acetaminophen, what is the most appropriate next step?

A Orthopedic surgery consult
B Prednisone taper
C Corticosteroid injection
D Shoes with a wide toe box

A

Answer: D

Explanation:
• Hallux rigidus (1st metatarsophalangeal - MTP - joint osteoarthritis due to wear and tear of the joint space) is best treated with acetaminophen/NSAIDs, and other conservative measures such as soft insoles and shoes with a wide/high toe box to accommodate the toes better.
• Joint injection and surgery would be last resort options once conservative measures are exhausted and not effective.
• Oral prednisone can be used for severe pain, but in general the goal would be to avoid a systemic solution to a very focal problem to avoid systemic side effects.

116
Q

A 39 year-old female presents to your clinic with complaints of right knee pain and swelling. She has had gradual onset of this pain over the past 6 weeks without trauma. She owns a house cleaning business. On exam, McMurray and Lachman tests are negative. There is focal swelling of the skin over the right patella compared to the left. Palpation of this swelling reproduces her pain. What is the most likely diagnosis?

A Retropatellar bursitis
B Prepatellar bursitis
C Deep infrapatellar bursitis
D Superficial infrapatellar bursitis

A

Answer: B

Explanation:
• Prepatellar bursitis is inflammation of the bursal sac between the patella and the skin.
• Infrapatellar bursitis is swelling of either the superficial (just superficial to the patellar tendon) or deep (just deep to the patellar tendon) infrapatellar bursae.
○ Clergyman’s Knee (= infrapatellar bursitis) may be from overuse, spending lots of time on your knees, an injury or underlying knee condition.

117
Q

A 76 year-old male presents to you with the chief complaint of left foot pain. He denies numbness, tingling, or weakness. On exam you note the presence of a mallet toe. How would you describe the position of a mallet toe? MTP: metatarsophalangeal joint. PIP: proximal interphalangeal joint. DIP: distal interphalangeal joint.

A MTP normal, PIP flexion, DIP normal
B MTP normal, PIP normal, DIP flexion
C MTP normal, PIP normal, DIP extension
D MTP normal, PIP flexion, DIP extension

A

Answer: B

Explanation:
• Mallet toe is a toe demonstrating MTP normal, PIP normal, and DIP flexion.
○ It is typically due to shoes being too small/tight, or trauma.
○ Treatment involves proper footwear with enough room to accommodate the toes, in rare cases surgery.
• Hammer toe is a toe demonstrating MTP hyperextension, PIP flexion, and DIP extension.
• Claw toe is a toe demonstrating MTP hyperextension, PIP & DIP flexion.

118
Q
In evaluating a patient with pain, you decide to squeeze her left tibia together by wrapping your hands around the medial and lateral borders of the tibia and squeezing. The purpose of this test is to diagnose which of the following conditions?
AAchilles tendon rupture
BTibial plateau fracture
CMaisonneuve fracture
DShin splints
A

Answer: D

Explanation:
• Squeezing the tibia and reproducing a patient’s pain is indicative of shin splints (medial tibial stress syndrome).
• One may also perform a squeeze test to examine a high ankle sprain, but X-rays would be the best test for Maisonneuve fracture (a complication of a high ankle sprain), not physical exam.

119
Q
A 60 year-old female with history of systemic lupus erythematosus (SLE) presents with gradual onset left groin pain. She denies history of trauma, weakness, numbness, or tingling. You astutely choose to order hip x-rays, which reveal a linear zone of lucency in the subchondral bone of the femoral head, as well as osteopenia of the femoral head. What is the most appropriate next step in management?
AOrthopedic surgery consult
BRheumatology consult
CPhysical therapy
DMRI arthrogram of hip
A

Answer: A

Explanation:
• Avascular necrosis (AVN) of the femoral head occurs with gradual onset groin pain secondary to poor, severely decreased blood flow to the femoral head, frequently due to congenital causes (Legg-Calve-Perthes disease), chronic corticosteroid usage, alcohol, and SLE (lupus).
○ This patient has lupus and is at risk for chronic corticosteroid usage as well because of this.
○ AVN of the femoral head requires orthopedic surgery consultation. X-rays are useful as initial imaging modality for AVN of the hip, and can be followed up with MRI.
• Proximal femoral stress fracture typically occurs in underweight athletes training for long distance-running with other risk factors such as amenorrhea, disordered eating, osteoporosis.
• Slipped capital femoral epiphysis (SCFE) occurs typically in obese adolescent males.
• Intracapsular femur fracture typically occurs with trauma.
• MR arthrogram is useful if you suspect a labral tear.

120
Q
118. Which of the following is not a major risk factor for achilles tendonopathy?
AHypertension
BAge
COverpronation
DTight heel cords

Answer: A

Explanation:
• Risk factors for achilles tendon injuries generally include tight achilles tendons, overpronation, age, and overtraining.

A
121
Q
Which of the following is not a major risk factor for achilles tendonopathy?
AHypertension
BAge
COverpronation
DTight heel cords
A

Answer: A

Explanation:
• Risk factors for achilles tendon injuries generally include tight achilles tendons, overpronation, age, and overtraining.

122
Q
A 24 year-old left-footed professional soccer player experiences sudden onset left groin pain while attempting to kick the ball. She denies numbness, tingling, or weakness. On exam, she has pain to palpation of the left anterior groin, and pain with resisted left hip flexion. X-rays demonstrate nondisplaced bony fragmentation of the anterior inferior iliac spine (AIIS). What is the most appropriate next step?
ARICE
BPT
CUrgent orthopedic surgery consult
DDiagnostic ultrasound scan
A

Answer: A

Explanation:
• This patient presents with hip flexor strain (in this case rectus femoris due to the AIIS abnormalities - rectus femoris originates from AIIS).
• This can be caused by forceful hip flexion, especially during the eccentric portion of the movement.
• Soccer players and people who kick things (e.g. football punters or placekickers) are at risk.
• Rest, ice, compression, elevation (RICE) is important whether or not an avulsion fracture is present.
• The vast majority of cases of AIIS avulsion fractures do not require surgery.

123
Q
An 87 year-old female with a history of right MCA stroke resulting in left hemiparesis presents to your outpatient clinic in follow-up. She notes ongoing left shoulder pain since her stroke, left shoulder swelling, and occasional excessive sweating over her shoulder. On examination, when you initially touch her shoulder she recoils in severe pain. What is the best next diagnostic step?
ATriple phase bone scan
BStellate ganglion blockade
CLeft shoulder MRI
DLeft shoulder x-ray
A

Answer: B

Explanation:
• This patient is presenting with complex regional pain syndrome type 1 (CRPS 1), which can occur after a stroke.
• It is characterized by pain out of proportion to exam findings, excessive sweating of skin, local skin and hair atrophy, vasomotor changes to the involved body region, edema, and allodynia.
• The best diagnostic and therapeutic test for CRPS 1 is a stellate ganglion blockade.

124
Q
The function of the tibialis anterior is which of the following?
ADorsiflexion and eversion
BDorsiflexion and inversion
CPlantarflexion and eversion
DPlantarflexion and inversion
A

Answer: B

Explanation:
• Tibialis anterior’s function is dorsiflexion and inversion of the ankle.

125
Q
A 57 year-old female presents to your clinic in follow-up for right knee pain. She has tried and failed physical therapy and NSAIDs. Diagnostic ultrasound scan reveals a hypoechoic mass in the posterior knee, located between the semimembranosus and medial head of the gastrocnemius. What is the most appropriate next step?
ABracing
BColor doppler ultrasound
CAspiration and steroid injection
DSurgery
A

Answer: B

Explanation:
• A refractory Baker cyst is described here. Aspiration with steroid injection under ultrasound guidance is an appropriate intervention.
• However, this should only be performed after color doppler over the hypoechoic mass is demonstrated to be negative.
• Positive color doppler flow over the mass would indicate that this is NOT a cyst, but instead a vascular malformation such as an aneurysm (e.g. popliteal artery aneurysm).
• Aspiration and steroid injection would not be appropriate for an aneurysm.

126
Q
A 39 year-old male with no prior medical history presents with gradual onset right groin pain. He denies trauma, weakness, numbness, or tingling. He is a former professional baseball player. On exam, log roll is negative. There is no tenderness to palpation about the groin. With the patient supine, flexing the hip to 90 degrees, adducting beyond midline, and internally rotating the femur reproduces the patient’s groin pain. Which of the following is the most likely diagnosis?
AFemoroacetabular impingement (FAI)
BFemoroacetabular osteoarthritis
CPiriformis strain
DHip flexor strain
A

Answer: A

Explanation:
• Younger patients with significant athletics history can develop abnormal contact between the femur and acetabulum during ROM, causing groin pain during activity.
• This abnormal contact is the result of CAM and/or Pincer lesions developing on the hip joint.
• CAM lesion is essentially a large outgrowth of bone at the head/neck region of the femur.
• Pincer lesion is an outgrowth of bone on the acetabulum.
• Either can cause abnormal impingement of the hip during range of motion.
• Physical exam maneuvers for femoroacetabular impingement (FAI) include anterior and lateral hip impingement tests.
• Anterior hip impingement test is performed with the hip in adduction beyond midline, flexion to 90 degrees, and internal rotation, and can reproduce FAI and the patient’s pain.
• Lateral hip impingement test is performed with the hip in abduction, flexion, and external rotation, and may also reproduce the patient’s pain in FAI.
• Labral tears often occur secondary to FAI. This patient is too young to be suspicious of OA of the hip.
• Piriformis strain would produce buttock pain.
• Hip flexor strain would likely have some component of tenderness to palpation.

127
Q

A 24 year-old female is training for a marathon and develops gradual onset left shin pain. The pain occurs while running, but also with normal ambulation. She denies trauma. On exam, squeezing the tibia reproduces her pain. X-rays demonstrate a small left tibial stress fracture. What is the next best step?
AMake non-weight-bearing (NWB)
BOrthopedic surgeon referral
CMRI
DSoft shoe orthotic to relieve her pain with normal ambulation

A

Answer: A

Explanation:
• This patient with a tibial stress fracture (small, hairline fracture of the tibia due to excessive chronic forces on the tibia) who has pain even with normal ambulation needs to be made non-weight-bearing (NWB) on the painful leg.
• She should be gradually progressed to weight-bearing status as tolerated if her serial X-rays demonstrate adequate healing of the fracture, and then introduced to shoe orthotics, physical therapy, and correction of her possibly abnormal running gait. Her running mileage should be increased gradually as tolerated.
• Large fractures require orthopedic surgery referral.

128
Q
A 27 year-old female comes to your clinic with complaints of bilateral ankle pain. She says she has “gone clubbing” every night this week. On exam, you note a visible soft tissue swelling near her achilles tendons. What is this swelling otherwise called?
AMorton sign
BApophysitis
CHaglund deformity
DSever’s disease
A

Answer: C

Explanation:
• Someone who has ankle pain, posterior ankle swelling near achilles tendon, and has probably been wearing high heels (going out to the club repeatedly) is at risk for posterior ankle bursitis (retrocalcaneal or retroachilles bursitis) due to excessive friction forces upon the achilles tendon or calcaneus.
• These bursae expand and cause a soft tissue swelling called a Haglund deformity.
• Treatment is removal of the offending agent (i.e. wear more comfortable shoes), US-guided aspiration and steroid injection, and last resort of surgical removal.

129
Q
Clavicle fractures most commonly occur at which portion of the clavicle?
AThere is no difference in rates
BDistal ⅓
CMiddle ⅓
DProximal ⅓
A

Answer: C

Explanation:
• Clavicle fractures most commonly occur within the middle ⅓ of the clavicle.

130
Q
A 57 year-old female with a history of rheumatoid arthritis develops gradual onset bilateral arm pain with weakness. On exam her gait is unsteady and she exhibits hyperreflexia. Which of the following diagnoses is the most likely?
AGuillain-Barre syndrome
BC1-C2 subluxation
CCervical syrinx
DDemyelinating plaques
A

Answer: B

Explanation:
• Patients with rheumatoid arthritis are subject to its sequelae if the disease is not properly managed.
• One of these well known sequelae is C1-C2 subluxation (atlantoaxial subluxation) leading to cervical myelopathy.

131
Q

A 24 year-old female sustains a fall onto both of her elbows while her shoulders are in a flexed, adducted position, and experiences immediate right shoulder pain as a result. What is her most likely diagnosis?

A None of these answers is correct
B AC joint separation
C Posterior shoulder dislocation
D Anterior shoulder dislocation

A

Answer: C

Explanation:
• A flexed, adducted arm position places the shoulder in prime position to be susceptible to a posterior dislocation.
• An abducted, externally rotated shoulder is typically how anterior dislocations occur, which, indeed, occur far more frequently than posterior dislocations.

132
Q

A 55 year-old female presents to your clinic with the complaint of fever and elbow problems. She notes that for the past several weeks she has noticed her right elbow is “bigger” than the left elbow. She is a student and has been taking lots of college classes lately. She uses an elbow pad to no benefit. On exam, strength and sensation are intact. You notice that at the posterior elbow there is a very large swelling. It is compressible and warm. The patient notes some discomfort there and asks you to “do something”. What is the next best step?

A Aspiration
B RICE
C Physical therapy
D Surgical referral

A

Answer: A

Explanation:
• This patient presents with classic findings of olecranon bursitis, which is usually due to repetitive forces to the olecranon, such as grinding your elbow on a desk for several hours daily (art, desk work, etc.).
• A large pouch of fluid develops at the olecranon bursa because there is very little skin there to prevent it from expanding if it is inflamed.
• Treatment involves RICE, elbow pad, and considering aspiration.
• It is NOT usually an infectious process, but if the patient has a fever, then consider aspiration, culture, and antibiotics.
• This patient has failed conservative treatment, and aspiration of this bursa carries little risk if done properly. Caution the patient that there is a good chance it will recur following aspiration unless the irritant forces are removed that are causing the bursopathy in the first place.

133
Q

The process of lying the patient supine and abducting and externally rotating their shoulder while providing an anterior-directed force onto the humerus is intended to diagnose which of the following diseases?

A Posterior shoulder instability
B Glenohumeral joint arthritis
C Anterior shoulder instability
D Acromioclavicular joint disease

A

Answer: C

Explanation:
• This question describes the anterior apprehension test, which is a test for anterior shoulder instability. If the test is positive, the patient will experience sudden apprehension that their shoulder is about to dislocate.
• The AC joint is assessed using the Scarf test.
• GHJ arthritis shows generalized ROM loss and pain with crepitus.
• Posterior shoulder instability is diagnosed using the Kim or Jerk tests.

134
Q

All of the following muscles are knee flexors except which of the following?

A Sartorius
B Semitendinosus
C Biceps femoris
D Tensor fascia lata

A

Answer: D

Explanation:
• All of the following muscles cross the knee joint posteriorly, rendering them knee flexors, except for tensor fascia lata, which does not directly cross the knee joint, but does control the IT band distally and is a hip abductor and internal rotator.
• The IT band itself distally crosses the knee anteriorly and attaches to Gerdy’s tubercle of the tibia, and this does NOT provide a knee flexion force.

135
Q

In a lateral ankle sprain which of the following is the most likely injured ligament?

A Deltoid ligament
B Posterior talofibular ligament (PTFL)
C Anterior talofibular ligament (ATFL)
D Calcaneofibular ligament (CFL)

A

Answer: C

Explanation:
• The ATFL is the most commonly injured ligament in lateral ankle sprains, followed by the CFL.

136
Q

A 27 year-old female falls on her outstretched hand and develops sudden-onset hand pain. She is taken to the ED where x-rays reveal a scaphoid fracture in the distal ⅓ of the bone. What is the next best step?

A CT scan
B Orthopedic surgery consult
C MRI
D Thumb spica cast

A

Answer: D

Explanation:
• Scaphoid fractures present following trauma/FOOSH (fall on outstretched hand) and typically cause pain in the anatomic snuffbox.
• X-rays/CT scan/MRI may reveal a fracture of the scaphoid.
• If this fracture occurs within the proximal ⅓ of the bone, orthopedic surgery consultation is required.
• If within the distal ⅔ of the bone, thumb spica cast is appropriate.
• If initial X-rays are negative for fracture, but you still suspect a scaphoid fracture, place thumb in thumb spica cast and repeat X-rays in 2 weeks, as the fracture may reveal itself at that time.
• You can also consider CT/MRI if X-rays are negative.
• Pain in the anatomic snuffbox is a scaphoid fracture until proven otherwise, even if initial X-rays are negative.

137
Q

A 35 year-old female presents to your clinic with complaints of hand pain. She says her pain is worst in the morning, and improves later in the day. She denies numbness, tingling, or weakness. On exam, strength is intact. When attempting active flexion and extension of her digits in her hand, you notice her 3rd digit moves in a “jerky” fashion, and does not smoothly glide from flexion back into extension. When attempting to passively relax from finger flexion to finger extension, the 3rd digit becomes fixed in a flexed position; with activation of her finger extensors she is able to fully extend all digits; however, this causes her brief pain. What is the most likely diagnosis?

A Dupuytren contracture
B Stenosing tenosynovitis
C Jersey finger
D Mallet finger

A

Answer: B

Explanation:
• This patient exhibits classic findings of trigger finger, which is a stenosing tenosynovitis of one of the hand digits, causing thickening of the finger flexor tendon sheath, thus preventing the tendon and its sheath from smoothly gliding underneath the A1 pulley at the MCP joint.
• Finger lies in slight extension at the DIP relative to other fingers in the resting position. Radiographs may show a bony avulsion if present.
• This causes the finger to snap and catch when flexing/extending the finger, and it can cause pain.
• Treatment involves splinting/immobilizing the finger to resolve the inflammation, ice, NSAIDs, steroid injection into the tendon sheath, and finally surgery.

138
Q

Shoulder dislocations most commonly occur in which of the following directions?

A All of the listed directions in equal proportions
B Lateral
C Posterior
D Anterior

A

Answer: D

Explanation:
• Anterior shoulder dislocations are the most common, followed by posterior dislocations.

139
Q
Which of the following types of dermatomyositis is associated with cancer?
A4
B3
C2
D1
A

Answer: B

Explanation:
• Dermatomyositis type 3 is associated with cancer (e.g. breast/lung).

140
Q
A 40 year-old male is water-skiing when he feels a sudden pop with burning pain in his left posterior thigh. There is a large amount of discoloration on the skin of the posterior thigh. What is the most likely diagnosis?
AFemoral shaft fracture
BLumbar radiculopathy
CHamstring tear
DACL tear
A

Answer: C

Explanation:
• This patient presents with hamstring strain/tear. It is caused by eccentric stretching of the hamstrings against a hip-flexing and knee-extending force, exactly the type of forces encountered while water-skiing.
• An acute tear may be accompanied by extensive thigh bruising, as noted here.
• X-rays are appropriate to rule out ischial tuberosity avulsion fracture (where the hamstrings originate).
• Treatment typically involves RICE, NSAIDs, PT.

141
Q

A dashboard injury (striking the knees against the car dashboard in a car accident) is the typical mechanism of injury for which of the following structures?

A ACL
B PCL
C Popliteus muscle
D Oblique popliteal ligament

A

Answer: B

Explanation:
• Dashboard injury is the typical mechanism of action for PCL injuries. Typically the knee is flexed (thus placing the PCL in its most tense position) when sitting in a car, and the subsequent dashboard trauma while in this position enables the rupture of the PCL.

142
Q

52 year-old male with a history of thoracic paraplegia presents to your office with complaints of bilateral shoulder pain. He achieves mobility using a manual wheelchair. On physical examination, you document positive Neer, Hawkins, and Speed tests bilaterally. He is able to achieve full shoulder abduction with significant pain. What is your next best step?

A Start physical therapy
B Acetaminophen and ibuprofen
C Shoulder MRI
D Shoulder xrays

A

Answer: A

Explanation:
• Patients requiring manual wheelchairs for mobility very commonly develop accelerated glenohumeral joint arthritis, rotator cuff disease, and subacromial impingement/bursopathy in some cases.
• These need to be treated just like any other patient with musculoskeletal disease. This patient presents with positive exam findings suggestive of rotator cuff disease without complete tearing (as evidenced by his ability to achieve full ROM of his shoulders).
• The most appropriate first step is physical therapy to increase strength and function of the rotator cuff muscles and reduce pain. If this fails, shoulder X-rays would be reasonable. Prescribing pain medications alone without therapy will not treat the actual problem.

143
Q

A 52 year-old cyclist presents with 2 months of progressive wrist pain. He denies weakness, numbness, or tingling. He denies trauma. On exam you note focal tenderness over the dorsal wrist just ulnar to the anatomic snuffbox. What is the next best step?

A Wrist bracing
B Wrist CT scan
C Wrist X-rays
D Reassurance

A

Answer: C

Explanation:
• This patient presents with findings concerning for idiopathic avascular necrosis of the lunate (AKA Kienbock Disease).
• The anatomic snuffbox floor is the scaphoid bone; thus, just ulnar to that site will allow the examiner to palpate the lunate bone. Pain at this spot with or without trauma is concerning for lunate pathology.

X-rays are the most appropriate initial imaging modality for most bony diseases, including suspicion of AVN, as in this case. Orthopedic surgery consultation would be appropriate if you have suspicion of Kienbock Disease, with or without positive X-ray findings. MRI can supplement X-ray findings, or provide a diagnosis if X-rays are negative.

144
Q

A 60-year old female presents with bilateral shoulder pain and stiffness for the past 1 year along with headaches and occasional pain in her jaw. Labs reveal high ESR and CRP. What is the most appropriate initial treatment?

A High-dose corticosteroids
B Hydroxychloroquine
C Methotrexate
D Physical therapy

A

Answer: A

Explanation:
• Polymyalgia rheumatica (PMR) presents classically as this patient does. PMR is associated with temporal arteritis (giant cell arteritis). Thus, it is very important to screen these patients for temporal arteritis by asking about jaw pain (jaw claudication) and headaches.
• For Polymyalgia rheumatic, the lowest effective dose of Prednisone that controls their symptoms is indicated. Physical therapy is also appropriate in addition to Prednisone.
• For Temporal arteritis (this patient), high dose IV steroids are appropriate in order to prevent blindness.

145
Q

A 32 year-old male is playing tennis. In an attempt to impress his spouse, he tries to catch a 140 mph serve with his bare hand. Shockingly, this does not occur as intended, and he develops sudden-onset finger pain. After delivering this blistering serve and declaring that you swung with minimal effort, you rush over to examine his finger. On exam, you notice swelling of the distal interphalangeal joint (DIP) of his index finger. There is inability to actively extend the DIP. There is pain with varus stress testing of the DIP. What is the most appropriate next step?

A Diagnostic ultrasound scan of the DIP
B Orthopedics consult
C X-rays
D DIP and PIP immobilization splinting

A

Answer: C

Explanation:
• This patient has suffered a mallet finger (sudden flexion of the DIP causing extensor tendon rupture, which can often pull off a bony fragment along with the tendon, known as an avulsion fracture).
• Treatment involves immobilization splinting to allow the tendon to heal back onto the bone, but X-rays are also necessary to rule out avulsion fracture, which may require orthopedic surgery.

146
Q

Which of the following positions is optimal to prevent contracture in a burn patient?

A Hip extension
B Plantarflexion
C Thumb adduction
D Wrist extension

A

Answer: D

Explanation:
• Optimal patient positioning after a burn involves neck extension, shoulder abduction, forearm supination, wrist extension, MCP flexion, PIP and DIP extension, palmar abduction, hip neutral, knee extension, and ankle dorsiflexion.

147
Q

Which of the following juvenile rheumatoid arthritis (JRA) subtypes requires ophthalmology referral? RF: rheumatoid factor.

A Systemic
B Pauciarticular
C Polyarticular, RF-
D Polyarticular, RF+

A

Answer: B

Explanation:
• Pauciarticular JRA requires ophthalmology referral, as this subtype is associated with iridocyclitis/uveitis.

148
Q

You are performing a palpation-guided trigger finger injection. You advance the needle into the target location and ask the patient to flex the involved finger. With finger flexion, the syringe moves. Which of the following is the most appropriate next step?

A Abort the procedure, as the patient’s anatomy is prohibitive for injection
B Inject the medication, as you are in the correct location
C Advance the needle tip slightly, then repeat finger flexion
D Withdraw the needle tip slightly, then repeat finger flexion

A

Answer: D

Explanation:
• A palpation-guided trigger finger injection requires that the needle tip be advanced just into the tendon sheath, but NOT into the tendon itself.
• Thus, if the needle tip lies within the tendon itself, upon finger flexion the syringe and needle will move along with the tendon.
• The key here is to withdraw the needle tip slightly, so that the needle tip remains within the tendon sheath, but not within the tendon. If the needle tip lies in the tendon sheath alone, then upon finger flexion, the syringe will NOT move, indicating that the needle tip is in the correct location, and the injection may proceed.

149
Q

A 39 year-old male falls on his outstretched hand. He develops sudden-onset pain located in the region between his extensor pollicis brevis and longus tendons in his wrist. Wrist x-rays are negative for fracture. What is the most appropriate next step?

A Physical therapy
B Rest, ice, compression, elevation
C Orthopedic surgery consult
D Thumb spica cast and repeat x-rays in 2 weeks

A

Answer: D

Explanation:
• This patient presents with pain in the anatomic snuffbox following trauma. Scaphoid fractures present following trauma/FOOSH (fall on outstretched hand) and typically cause pain in the anatomic snuffbox.
• X-rays/CT scan/MRI may reveal a fracture of the scaphoid; if this fracture occurs within the proximal ⅓ of the bone, orthopedic surgery consultation is required.
• If within the distal ⅔ of the bone, thumb spica cast is appropriate.
• If initial x-rays are negative for fracture, but you still suspect a scaphoid fracture, place thumb in thumb spica cast and repeat x-rays in 2 weeks, as the fracture may reveal itself at that time.
• You can also consider CT/MRI if x-rays are negative.
• Pain in the anatomic snuffbox is a scaphoid fracture until proven otherwise, even if initial x-rays are negative.

150
Q

A patient develops lateral knee pain, worse with skiing downhill. He notes that running downhill also causes this pain. On exam, varus and valgus stress testing is negative, as is McMurray, Thessaly, Apley grind, Lachman, posterior drawer, patellar grind, and patellar excursion. There is no tenderness to palpation throughout the knee. He denies stiffness, swelling, numbness, tingling, or pain when going up or down stairs. What is the most likely diagnosis?

A Popliteus tendonitis
B Bursitis
C Patellar tendonitis
D Quadriceps tendonitis

A

Answer: A

Explanation:
• Lateral knee pain worse with running downhill or skiing downhill is a classic presentation of popliteus tendonitis.
• The popliteus muscle unlocks the knee and is located laterally within the knee joint.
• Treatment involves rest, ice, NSAIDs, and physical therapy.

151
Q

A 39 year-old male has just returned from a skiing trip. He presents to you with complaints of gradual-onset left knee pain ever since the trip. He developed aching knee pain without significant trauma. He denies numbness, tingling, or weakness. He notes he skied lots of moguls when he was at the resort. On exam, medial joint line palpation produces pain, as does valgus stress testing. Lachman test is negative. What is the most likely diagnosis?

A ACL tear
B Medial meniscus tear
C LCL tear
D MCL tear

A

Answer: D

Explanation:
• The MCL (medial collateral ligament) of the knee is the most commonly torn knee ligament out of all causes.
• The ACL (anterior cruciate ligament) is the most commonly torn knee ligament in sports.
• Yes, the patient was skiing. But in the absence of a positive anterior drawer or Lachman test, sudden-onset knee pain, or knee swelling, ACL tear is far less likely.
• MCL testing is positive in this patient, and recent history of knee stresses favors this diagnosis.

Medial meniscus tear is a possibility, but there is typically a history of locking/catching/buckling of the knee as well as knee swelling with this; in addition, MCL tear is more likely to occur than medial meniscus tear.

152
Q

A 55 year-old female presents with bilateral shoulder pain and stiffness over the past 6 months. Labs reveal elevated ESR and CRP. Which of the following additional symptoms is most likely present in this patient?

A Numbness/tingling
B Jaw pain
C Knee pain
D Dry mouth

A

Answer: B

Explanation:
• Polymyalgia rheumatica (PMR) presents classically as this patient does. PMR is associated with temporal arteritis (giant cell arteritis). Thus, it is very important to screen these patients for temporal arteritis by asking about jaw pain (jaw claudication) and headaches.
• For PMR the lowest effective dose of Prednisone that controls their symptoms is indicated.
• Physical therapy is also appropriate in addition to prednisone.

153
Q

Which part of the scaphoid bone has poor blood supply, rendering scaphoid fractures at that location susceptible to which feared complication?

A Proximal ⅓, radial artery dissection
B Proximal ⅓, avascular necrosis
C Distal ⅓, radial artery dissection
D Distal ⅓, avascular necrosis

A

Answer: B

Explanation:
• Scaphoid fractures present following trauma/FOOSH (fall on outstretched hand) and typically cause pain in the anatomic snuffbox.
• X-rays/CT scan/MRI may reveal a fracture of the scaphoid; if this fracture occurs within the proximal ⅓ of the bone, orthopedic surgery consultation is required.
• If within the distal ⅔ of the bone, thumb spica cast is appropriate.
• If initial x-rays are negative for fracture, but you still suspect a scaphoid fracture, place thumb in thumb spica cast and repeat x-rays in 2 weeks, as the fracture may reveal itself at that time.
• You can also consider CT/MRI if x-rays are negative.
• Pain in the anatomic snuffbox is a scaphoid fracture until proven otherwise, even if initial x-rays are negative.

154
Q

The purpose of the muscles within the lateral compartment of the lower leg is to perform which of the following actions?

A Plantarflexion and eversion
B Plantarflexion and inversion
C Dorsiflexion and eversion
D Dorsiflexion and inversion

A

Answer: A

Explanation:
• The fibularis longus and brevis are located within the lower leg’s lateral compartment along with the superficial fibular nerve which innervates these muscles. Their action is to cause plantarflexion and eversion of the ankle.

155
Q

A 29 year-old male presents in follow up for left tennis elbow. He has tried RICE, PT, acetaminophen/NSAIDs, and corticosteroid injection to only minimal benefit. It is summer time, and he is disappointed because he is looking forward to playing in an upcoming tournament in London. He wonders if there is anything else he can do. Which of the following is the most appropriate recommendation?

A Surgery
B TENS
C Needle tenotomy of common extensor tendon
D Repeat corticosteroid injection

A

Answer: C

Explanation:
• This patient has failed conservative treatments as well as one corticosteroid injection which provided NO benefit.
• At this point regenerative interventions may be considered, including needle tenotomy, platelet-rich plasma, prolotherapy, etc.
• Surgery would be a last resort.

156
Q

The area of hypovascularity of the achilles tendon (leading to tears in this region) is generally at which of the following locations on the tendon?

A Distal 2-6 cm of the tendon
B The musculotendonous junction
C Proximal 2cm of the tendon
D As the tendon attaches to the calcaneus

A

Answer: A

Explanation:
• The achilles tendon’s zone of hypovascularity is generally within the distal 2-6 cm of the tendon, predisposing this region to tendon tears due to the poor blood supply.

157
Q

The third extensor compartment of the wrist contains which of the following tendons?

A Extensor indicis proprius
B Extensor pollicis brevis
C Extensor pollicis longus
D Extensor digiti minimi

A

Answer: C

Explanation:
• The third extensor compartment of the wrist contains the EPL tendon.

158
Q

A 34 year-old female presents to your clinic with complaints of hand pain. She says her pain is worst in the morning, and improves later in the day. She denies numbness, tingling, or weakness. On exam, strength is intact. When attempting active flexion and extension of her digits in her hand, you notice her 3rd digit moves in a “jerky” fashion, and does not smoothly glide from flexion back into extension. When attempting to passively relax from finger flexion to finger extension, the 3rd digit becomes fixed in a flexed position; with activation of her finger extensors she is able to fully extend all digits; however, this causes her brief pain. NSAIDs and splinting have not provided relief. What is the next most appropriate step?

A Repeat splinting trial
B Occupational therapy
C Corticosteroid injection
D Surgical release

A

Answer: C

Explanation:
• This patient exhibits classic findings of trigger finger, which is a stenosing tenosynovitis of one of the hand digits, causing thickening of the finger flexor tendon sheath, thus preventing the tendon and its sheath from smoothly gliding underneath the A1 pulley at the MCP joint.
• This causes the finger to snap and catch when flexing/extending the finger, and it can cause pain.
• Treatment involves splinting/immobilizing the finger to resolve the inflammation, ice, NSAIDs, steroid injection into the tendon sheath, and finally surgery.

159
Q

You are examining a patient with right knee pain. You ask the patient to perform a right leg single leg squat while twisting on the right knee. What is the purpose of this test?

A To diagnose meniscus tears
B To diagnose MCL/LCL tears
C To assess balance and strength
D To diagnose knee osteoarthritis

A

Answer: A

Explanation:
• This question describes the Thessaly physical exam maneuver, a test for medial or lateral meniscus tears in the knee.

160
Q

Which of the following knee ligaments is the most commonly torn overall?

A ACL
B PCL
C MCL
D LCL

A

Answer: C

Explanation:
• The MCL (medial collateral ligament) is the most commonly torn overall knee ligament.
• The ACL is the most commonly torn knee ligament in sports.

161
Q

A 36 year-old male suffers a fall onto his outstretched hand. He has immediate shoulder disfigurement which is corrected by a bystander. In the ED, xrays of the shoulder reveal a Hill-Sachs lesion. This lesion can be described as which of the following?

A Kim lesion
B Proximal humeral displaced fracture
C Anterior labral tear
D Posterolateral humeral head compression fracture

A

Answer: D

Explanation:
• This patient has suffered an anterior shoulder dislocation.
• Xray evidence of a posterolateral humeral head compression fracture is known as a Hill-Sachs lesion, which is associated with anterior shoulder dislocations.
• MRI evidence of anterior labrum injury in the setting of shoulder disfigurement (= dislocation) is known as a Bankart lesion.
• A Hill-Sachs lesion and anterior labral tear are not the same thing.

162
Q

A 25 year-old female soccer player presents to your office with 1 month of groin pain. It is mildly improved with ibuprofen. She denies numbness or tingling, as well as any bowel or bladder dysfunction. She does endorse that she feels weak in her legs. On exam you note 5/5 strength in the lower limbs, as well as intact sensation and normal reflexes. Bilateral FABERE causes pain in the ipsilateral groin. There is tenderness to palpation of the bilateral very proximal medial thighs. There is no pain with resisted lower limb adduction or abduction. What is the next best step?

A RICE
B Corticosteroid injection
C MRI
D Orthopedic surgery consult

A

Answer: A

Explanation:
• This patient presents with osteitis pubis, which is pubic symphysis inflammation due to adductor overuse; this can be seen in sports, especially those involving use of the adductor muscles of the lower limb.
• It presents as vague groin pain worse with resisted adduction (though not always!) and direct palpation of the proximal adductors or pubic symphysis.
• Xrays can be useful which will show changes in the pubic bone, sometimes widening of the symphysis itself.
• MRI can be useful but is not the first imaging choice.
• Treatment involves first RICE (rest, ice, compression, elevation), NSAIDs, then PT, and you may consider corticosteroid injection into the symphysis in refractory cases.

163
Q

A 24 year-old female complains of gradual onset worsening right hip pain. She is currently training for a marathon. She denies weakness, numbness, and tingling. X-rays reveal a cortical lucency along the superior/lateral femoral neck. There is no pain during normal ambulation. What is the most appropriate next step in management?

A Physical therapy
B Make right leg non-weight-bearing
C Orthopedic surgery consult
D Closed reduction and casting

A

Answer: C

Explanation:
• Tension-side proximal femur stress factor:
○ Risk factors include low BMI, poor nutrition, amenorrhea, osteoporosis.
○ Tension-side fractures (fractures along superior/lateral femoral neck)
○ Tension-side fractures are less stable and require orthopedic surgery consult for ORIF.
• Compression-side fractures (on inferior/medial side of femoral neck) are more common

Can be treated with non-weight-bearing for a period of time, then gradual weight-bearing as tolerated, then gradually increase activity.

164
Q

The type of muscle fiber that purely utilizes glycolysis, not aerobic respiration, for energy is:

A B and C
B Type IIb
C Type IIa
D Type I

A

Answer: B

Explanation:
• Type I muscle fibers are slow twitch, “red” fibers that utilize oxidative phosphorylation as part of aerobic respiration.
○ These are the muscles primarily utilized in cardiovascular endurance exercises such as running a marathon.
• Type IIa fibers are fast twitch, “white” muscle fibers that are anaerobic and utilize glycolysis primarily for energy.
○ Type IIa fibers also do have some degree of aerobic capacity.
• Type IIb fibers are purely glycolytic and have no aerobic capacity.
○ These are used chiefly in powerful fast twitch activities, such as sprinting or powerlifting.

165
Q

A 24 year-old long distance runner complains of gradual onset bilateral anterior knee pain. His pain is worse when descending stairs. He recently watched a movie, and afterwards noticed that his knees were stiff when leaving the cinema. On exam, there is no tenderness to palpation of the knees. When compressing and pushing the patella inferiorly, you ask the patient to contract their quadriceps; this elicits pain and a grinding sensation. What is the mechanism of this patient’s most likely disorder?

A Weak vastus lateralis
B Meniscal tearing
C Abnormal patellar tracking
D Tibiofemoral cartilage degeneration

A

Answer: C

Explanation:
• This patient presents with classic patellofemoral pain syndrome (PFPS).
• This is abnormal patellar tracking due to muscle weakness and imbalances; specifically this involves vastus lateralis and IT band tightness in combination with vastus medialis weakness.
• Treatment involves vastus medialis strengthening with vastus lateralis and IT band stretching, along with hip girdle strengthening for stability.
• Sometimes patellar knee sleeves are used to assist with the proprioception of proper patellar tracking, as is kinesiotaping.
• The patellar grind test is described here, which is a test for PFPS.\If rest, physical therapy, NSAIDs, and bracing do not improve the pain, MRI and surgery may be considered.
• Note: typically this is the muscular pattern of weakness and tightness for PFPS, but it is possible to have a weak vastus lateralis and tight vastus medialis and perform essentially the opposite PT as for typical PFPS.

166
Q

A 58 year-old male presents with sudden-onset right knee pain and swelling. He denies trauma. Synovial fluid analysis reveals crystals with a positive birefringence pattern. What is the pathophysiology of this disease?

A Calcium pyrophosphate deposition
B Elevated acute phase reactants
C Abnormal red blood cell morphology
D Elevated uric acid

A

Answer: A

Explanation:
• Pseudogout causes sudden onset pain (usually in the knee) due to calcium pyrophosphate crystal deposition (also called chondrocalcinosis).
• Synovial fluid analysis reveals crystals with a positive birefringence pattern.
• Treatment is indomethacin, colchicine, and corticosteroids for acute attacks.

167
Q

A 57 year-old female presents to your clinic with the complaint of fever and elbow problems. She notes that for the past several weeks she has noticed her right elbow is “bigger” than the left elbow. She is a student and has been taking lots of college classes lately. She uses an elbow pad to no benefit. On exam, strength and sensation are intact. You notice that at the posterior elbow there is a very large swelling. It is compressible and warm. You diagnose olecranon bursitis. The patient notes some discomfort there and asks you to “do something”. You perform in-office aspiration and notice immediate reduction in size of the bursa. You caution her that:

A She should change career goals
B She will ultimately require surgery
C It is likely to recur
D The bursa is likely to become infected

A

Answer: C

Explanation:
• Olecranon bursitis has a good chance of recurrence once aspirated.
• However, aspiration is a reasonable intervention for this disease if the patient fails conservative treatment of RICE and elbow padding.

168
Q

A 47 year-old female with longstanding rheumatoid arthritis (RA) is currently taking methotrexate and ibuprofen. Her disease, while improved, has unfortunately been somewhat resistant to these treatments. She recently completed a course of physical therapy and continues her home exercise program daily. Which of the following is the next best step?

A Bracing
B Orthopedic surgery consult for joint fusion
C Initiate corticosteroid therapy
D Initiate prescription strength NSAID

A

Answer: C

Explanation:
• DMARDs (disease-modifying antirheumatic drugs) are first-line treatment in RA.
• NSAIDs may be used adjunctively.
• Failing these, oral corticosteroids may be added for chronic disease suppression.
• Orthopedic surgery is a last resort.

169
Q

A 23 year-old male is playing American football, attempting to tackle his opponent. He experiences sudden hand pain in the process. On exam, when pinning down the proximal interphalangeal joints (PIPs) of digits 2-5, active flexion of the distal phalangeal joints (DIPs) occurs in all fingers except the index finger. What is the most appropriate next step?

A MRI
B DIP immobilization splinting
C X-rays first, then orthopedic surgery consult if necessary
D X-rays and orthopedic surgery consult

A

Answer: D

Explanation:
• This patient exhibits classic findings of Jersey Finger (flexor digitorum superficialis or profundus avulsion off the respective interphalangeal joint).
• This occurs due to sudden hyperextension of the PIP or DIP, thus tearing the flexor tendon off the joint.
• This commonly occurs as a result of getting one’s finger caught in an opponent’s jersey during sports.
• Treatment is immediate x-rays to rule out avulsion fracture, and orthopedic surgery consultation for surgical reattachment of the flexor tendon onto the appropriate bone.

170
Q

Which of the following is an example of an isotonic exercise?

A Planks
B Bench press
C Pushing against a wall
D Rowing

A

Answer: B

Explanation:
• In isometric exercise, NO movement occurs throughout the entire exercise.
• In isotonic exercise, the weight (tone) being applied to a muscle or joint is the same throughout the entire exercise (e.g. biceps curls - the dumbbell weighs the same, and, thus, imparts the same forces onto the arm for the entire exercise).
• In isokinetic exercise, the movement and rhythm of the exercise is the same throughout the entire exercise (e.g. cardiovascular machines such as the elliptical, cycling, rowing).

Isotonic Exercise
• Isotonic exercise is movement that requires muscles to resist weight over a range of motion, causing a change to the length of the muscle. We usually think of muscles shortening in isotonic exercise, as when you lift a dumbbell for a bicep curl or rise into a sit-up. This is called concentric muscle contraction.
• Aerobic exercises like walking, running, hiking, swimming, skiing, and dancing are all considered isotonic exercise. So are resistance training exercises that involve movement, such as squats, pushups, pull ups, bench presses, deadlifts, and bicep curls.

Isometric Exercise
• Isometric exercise involves remaining in a static position while engaging the muscles. The joint doesn’t move, and the muscle neither lengthens or shortens, but the muscle’s tendon is activated.
• Stationary exercises such as wall sits, planks, bridges, hollow-body holds, are isometric.

171
Q

The PCL (posterior cruciate ligament of the knee) originates on the femur, and its fibers run in which of the following directions on its way towards the tibia?

A Postero-infero-laterally
B Antero-supero-medially
C Antero-infero-medially
D Postero-infero-medially

A

Answer: A

Explanation:
• The PCL runs posteroinferolaterally from the femur toward the tibia. It tenses with knee flexion. It limits translation of the tibia in the posterior direction.

172
Q

A 33 year-old male is involved in a traumatic accident. He develops left elbow pain shortly after. He denies weakness, numbness, or tingling. On exam, strength is intact. Cozen test is negative. When extended, there is valgus angulation to the elbow. Applying a medially directed force to the elbow joint while stabilizing the arm and forearm reproduces the patient’s pain. What is the most likely diagnosis?

A Radial collateral ligament sprain
B Ulnar collateral ligament sprain
C Golfer’s elbow
D Tennis elbow

A

Answer: B

Explanation:
• Valgus stress test: reproduction of pain with this is diagnostic for ulnar collateral ligament sprain.
• Cozen test (resisted wrist extension) is a test for tennis elbow, and is negative in this case.
• A varus stress test would be diagnostic for radial collateral ligament sprain if positive.
• Golfer elbow (medial epicondylitis) pain would be reproduced with resisted wrist flexion.

173
Q

A 30 year-old male develops left knee pain and swelling associated with fever. Synovial fluid analysis shows 130k WBCs with 97% neutrophils. What is the most likely etiology for this disease?

A Staph aureus
B Calcium pyrophosphate deposition
C Uric acid deposition
D Neisseria gonorrhea

Answer: D

Explanation:
• Septic arthritis in adults is typically due to N. gonorrhea infection.
• Synovial fluid analysis in septic arthritis reveals > 100k WBCs and overwhelming neutrophil (PMN) predominance.
• Septic arthritis in children is typically due to staph aureus.
• Crystal arthropathy (gout, pseudogout) would not show these synovial fluid findings (there would be < 100k WBCs and around 75% neutrophils).
A 30 year-old male develops left knee pain and swelling associated with fever. Synovial fluid analysis shows 130k WBCs with 97% neutrophils. What is the most likely etiology for this disease?

A Staph aureus
B Calcium pyrophosphate deposition
C Uric acid deposition
D Neisseria gonorrhea

A

Answer: D

Explanation:
• Septic arthritis in adults is typically due to N. gonorrhea infection.
• Synovial fluid analysis in septic arthritis reveals > 100k WBCs and overwhelming neutrophil (PMN) predominance.
• Septic arthritis in children is typically due to staph aureus.
• Crystal arthropathy (gout, pseudogout) would not show these synovial fluid findings (there would be < 100k WBCs and around 75% neutrophils).

174
Q

The conjoint hamstring tendon contains which tendons?

A Semitendinosus, biceps femoris, semimembranosus
B Semitendinosus, biceps femoris
C Semimembranosus, biceps femoris
D Semimembranosus, semitendinosus

A

Answer: B

Explanation:
• The conjoint hamstring tendon on the ischium houses both semitendinosus and biceps femoris.
• The semimembranosus originates more laterally than the other two hamstring tendons mentioned (somewhat paradoxically, as it ends up being the most medial hamstring muscle as you move distally).

175
Q

A 34 year-old female presents with 4 months of gradual onset low back pain. She denies trauma. On exam, flexion, abduction, and external rotation of the left lower limb causes right low back pain. What is the name of this maneuver?

A Stinchfield
B FABER
C Yeoman
D Gaenslen

A

Answer: B

Explanation:
• FABER maneuver: a sacroiliac joint test
• A positive test produces contralateral low back pain.

176
Q

A 36 year-old female presents to your neurotoxin clinic for evaluation of botulinum toxin for her cervical dystonia. On examination, her head is rotated to the right, and her left ear is tilted toward her left shoulder. What muscles are implicated in this scenario?

A Right sternocleidomastoid, left splenius capitis, left levator scapula
B Left sternocleidomastoid, left splenius capitis, left levator scapula
C Bilateral sternocleidomastoid (SCM)
D Bilateral splenius capitis and spinal erectors

A

Answer: B

Explanation:
• This patient with cervical dystonia presents with left torticollis, a type of cervical dystonia.
• Left torticollis results in dystonic contraction of the left SCM, resulting in head rotation to the right.
• In addition, the ipsilateral splenius capitis and levator scapula are often involved, resulting in head tilting toward the ipsilateral ear.
• Remember, contracting a given SCM results in head turning to the opposite side.

177
Q

During an ultrasound-guided injection, you notice that the needle appears as a bright white dot on the screen. Which of the following injection approaches is the physician using?

A Long axis
B In-plane
C Short axis
D Out-of-plane

A

Answer: D

Explanation:
• Short and long axis terminology describes the view of the target structure, not the approach of the needle.
• In short axis, a structure is viewed in “cross section”.
• In long axis, the structure is viewed along its entire length, and a cross-sectional view is NOT obtained.
• In-plane needle injection technique involves the needle length being parallel to the ultrasound transducer, and thus, the entire needle can be visualized underneath the skin.
○ The needle is viewed in long axis.
• Out of plane technique involves the needle being advanced perpendicular to the ultrasound transducer, and, thus, only a small (dot-sized) portion of the needle is viewed at any given moment on the screen.
○ The needle is viewed in short axis.

178
Q

The extensor digitorum attaches to which of the following structures?

A None of these answers is correct
B Both distal and proximal phalanges
C Proximal phalanx
D Distal phalanx

A

Answer: B

Explanation:
• The extensor digitorum muscle attaches to a thick piece of tissue called the extensor expansion, which pulls on both the DIPs and PIPs, causing extension of both.

179
Q

A 51 year-old male presents to your clinic with the complaint of fever and elbow problems. He notes that for the past several weeks he has noticed his right elbow is “bigger” than the left elbow. He is a student and has been taking lots of college classes lately. On exam, strength and sensation are intact. You notice that at the posterior elbow there is a very large swelling. It is compressible and warm. The patient denies pain. He asks what he should do. What is the next best step?

A Aspiration and culture
B Surgery
C RICE
D Elbow pad

A

Answer: A

Explanation:
• Olecranon bursitis: usually due to repetitive forces to the olecranon, such as grinding your elbow on a desk for several hours daily (art, desk work, etc.)
• A large pouch of fluid develops at the olecranon bursa because there is very little skin there to prevent it from expanding if it is inflamed.
• Treatment involves RICE, elbow pad, and considering aspiration.
• It is NOT usually an infectious process, but if the patient has a fever (as this patient does), then consider aspiration, culture, and antibiotics.

180
Q

In a Lisfranc foot sprain, the involved ligament connects which two bones?

A Medial cuneiform and 2nd metatarsal
B Middle cuneiform and 1st metatarsal
C Cuboid and 3rd metatarsal
D Navicular and 3rd metatarsal

A

Answer: A

Explanation:
• The Lisfranc ligament connects the medial cuneiform bone to the 2nd metatarsal and is a common ligament sprain in athletics.

181
Q

You are examining a patient with right knee pain. You ask the patient to perform a right leg single leg squat while twisting on the right knee. What is the name of this test?

A Apley Grind
B Thessaly
C McMurray
D Wrestler’s test

A

Answer: B

Explanation:
• Thessaly physical exam maneuver: a test for medial or lateral meniscus tears in the knee.

182
Q

You are examining a patient and decide to perform the Schober test. What does this exam test for?

A Inorganic sources of pain
B Hip joint disease
C Sacroiliac joint disease
D Lumbar range of motion

A

Answer: D

Explanation:
• The Schober test demonstrates limited lumbar flexion.
• It is performed by having the physician mark a spot on the skin roughly at the L5 level, then mark a spot 10cm superior to this spot and 5cm inferior to it.
• With forward lumbar flexion, these spots should separate, and the distance between them (normally 15cm) should increase to greater than 20cm.
• In ankylosing spondylitis (AS) patients this lumbar flexion will be so limited that the distance may only increase by under 5cm, indicating a positive test in support of the AS diagnosis.

183
Q

Which of the following structures forms the lateral border of Guyon’s canal?

A None of these answers is correct
B Hypothenar muscles
C Hook of the Hamate
D Pisiform

A

Answer: C

Explanation:
• Hook of the Hamate forms the lateral (radial) border of Guyon’s canal, while the pisiform forms the medial border.
• The hypothenar muscles form the floor of Guyon’s canal along with the flexor retinaculum.
• The ulnar nerve can become entrapped within Guyon’s canal, producing an ulnar neuropathy at the wrist. A hook of the Hamate fracture can cause ulnar neuropathy at the wrist.

184
Q

You ask your patient to abduct and externally rotate their shoulders to 90 degrees and then begin opening and closing their hands over a 3-minute period. This test is known by which name, and evaluates which condition?

A Roos, lower trunk brachial plexopathy
B Cyriax, ulnar neuropathy at the elbow
C Wright, vascular claudication
D Adson, venous congestion

A

Answer: A

Explanation:
• Roos test: assesses neurogenic thoracic outlet syndrome, which itself is a lower trunk plexopathy
• A positive test is indicated by reproduction of the patient’s symptoms in the upper limb (pain, numbness, tingling in a C8-T1 pattern).

185
Q

A 12 year-old male presents to your sports medicine clinic for evaluation of left shoulder pain radiating down towards the left elbow. He is a baseball pitcher and his team is in the middle of a tournament at the end of the season. He denies weakness, numbness/tingling, and history of trauma. He has noticed progressive shoulder pain as the season has progressed, worse after a game in which he pitches. On exam, he has a negative scarf, Neer, and Hawkins test. He has pain with empty can and O’Brien maneuvers. He is tender to palpation over the mid-upper arm. What is going to be your most likely intervention in this case?

A Reassurance
B Rest
C Surgery
D Injection

A

Answer: B

Explanation:
• This patient is suffering from a humeral stress fracture, brought on by excessive pitching. These patients typically have pain with resisted shoulder movements, hence the pain with empty can and O’Brien tests.
• These patients are tender to palpation over the fracture site.
• X-rays are the best initial step, followed by rest for several weeks.
• You must prescribe rest to allow the stress fracture to heal and prevent it from becoming a worse, displaced fracture.
• Injection would NOT be appropriate, as there is NO defined target, and deciding to inject into a stress fracture would NOT be safe.
• Surgery is usually NOT required unless the stress fracture is shown to be significantly displaced on X-ray.

186
Q

Which of the following is the primary structure that maintains the humeral head within the glenoid fossa?

A The glenoid labrum
B The inherent bony anatomy
C The glenohumeral ligaments
D The rotator cuff

A

Answer: D

Explanation:
• Rotator cuff is chiefly what maintains proper contact of the humeral head within the glenoid fossa.
• When these muscles are weak (e.g. stroke) the humeral head is far more likely to sublux out of the glenoid fossa.
• The other structures listed are secondary to the rotator cuff in this role.

187
Q

A 31 year-old male is playing tennis. In an attempt to impress his girlfriend, he tries to catch a 130 mph serve with his bare hand. Shockingly, this does not occur as intended, and he develops sudden-onset finger pain. After delivering this blistering serve and declaring that you swung with minimal effort, you rush over to examine his finger. On exam, you notice swelling of the distal interphalangeal joint (DIP) of his index finger. There is pain with varus stress testing of the DIP. X-rays are negative for acute fracture. What is the most appropriate initial step in management?

A Reassurance / play the next point “like a man”
B Dynamic varus stress testing under ultrasound
C Orthopedic surgery referral
D Immobilization splinting for the DIP

A

Answer: D

Explanation:
• This patient has sustained a radial collateral ligament injury/sprain/tear of his index finger DIP joint.
• X-rays have ruled out fracture of the finger/hand, so the most appropriate step is to allow the ligament to heal by using either buddy tape or an extension splint for the finger the prevents flexion, varus, and valgus forces about the DIP.
• This is not a surgical issue at this point.

188
Q
  1. A 59 year-old female presents with left knee pain of gradual onset without a history of trauma. She points to the medial knee in a vague, large circle when you ask where the pain is. On exam strength, sensation, and reflexes are intact. Valgus stress testing is negative. Varus stress testing causes medial knee pain. There is pain with resisted hamstring strength. Palpation of the medial tibia below the knee elicits concordant pain. The left medial tibia appears swollen compared to the right. What is the most likely diagnosis?

A Pes anserine bursitis
B Hamstring tendonitis
C LCL sprain
D Medial meniscus tear

A

Answer: A

Explanation:
• This patient depicts classic findings of pes anserine bursitis.
• Pes anserine consists of sartorius, gracilis, and semitendinosus which insert onto the medial tibia below the knee.
• There is also a bursa here which can become inflamed and distended.

189
Q

A 39 year-old female presents to your movement disorders clinic for neck pain. Examination reveals her right ear to be tilting toward her right shoulder with a forward-facing gaze. How would you describe her neck posture?

A Left torticollis
B Right torticollis
C Left laterocollis
D Right laterocollis

A

Answer: D

Explanation:
• Laterocollis is the ipsilateral abnormal posture of “ear to shoulder” without an SCM (sternocleidomastoid) contraction component.
○ In laterocollis, the muscles implicated are ipsilateral splenius capitis, scalenes, and levator scapula.
• If the SCM were to be involved, this would result in head rotation toward the opposite side of the SCM, and we would characterize this as having a torticollis component.

190
Q

A 62 year-old male with a history of diabetes mellitus presents to your clinic with complaints of right wrist pain. Finkelstein test is positive. On exam you notice a flexion-biased position of the 3rd finger while the remaining digits are extended. Passive range of motion of the 3rd digit is difficult. What is the most likely diagnosis related to this finding?

A Jersey finger
B Dupuytren contracture
C Trigger finger
D Osteoarthritis

A

Answer: B

Explanation:
• Patients with seizures, diabetes, or alcoholism are most at risk for Dupuytren contracture, which is a thickening of the palmar fascia that causes painless swelling and flexion contracture of a digit, usually the 3rd digit (ring finger).
• Trigger finger would be moveable with discernible “jerking” of the finger as it flexes and extends; it would NOT simply be fixed in place and difficult to move.
• OA would NOT generally cause a contracture of a single joint in the fingers while sparing the others.
• Jersey finger is an avulsion of the flexor digitorum profundus tendon off the distal phalanx, leading to inability to actively flex the DIP of the involved digit.
• The positive Finkelstein test (indicated De Quervain tenosynovitis - likely the cause of this patient’s wrist pain) is a red herring in this question.

191
Q

You are examining a patient with right knee pain. The patient is seated with the hip flexed and knee extended. You flex their knee beyond 90 degrees, and then let it drop into extension via gravity. What is the name of this test?

A Thessaly
B McMurray
C Apley
D Bounce Home

A

Answer: D

Explanation:
• Bounce Home physical exam maneuver: a test for medial or lateral meniscus tears in the knee.

192
Q

A 65 year-old female presents to your clinic with complaints of 4 months of left knee pain. She has tried physical therapy and acetaminophen to no benefit. A corticosteroid injection into the knee provided 3 weeks of relief. Left knee x-rays demonstrate severe medial compartment narrowing, reactive sclerosis, osteophytic changes, and subchondral cysts. What is the next most appropriate step?

A Knee replacement
B PRP (platelet rich protein) injection
C Reassurance
D Knee MRI

A

Answer: A

Explanation:
• This patient has failed conservative and interventional measures for her severe knee OA, and would be a good candidate for total knee replacement.
• Regenerative injections such as PRP are most useful before arthritis becomes severe, and likely would not provide significant lasting benefit in her case.
• Knee MRI would likely provide further information on how bad the knee OA is, and possibly diagnose a meniscal tear, all of which would only further support our conclusion that she would likely benefit from a knee replacement.

193
Q

Which of the following is the most common type of ankle sprain?

A Posterior
B Anterior
C Lateral
D Medial

A

Answer: C

Explanation:
• Lateral ankle sprains are by far the most common type of ankle sprain.
• It is a hyperinversion injury that injures the ATFL, CFL, and PTFL ligaments.
• ATFL: anterior talofibular ligament. CFL: calcaneofibular ligament. PTFL: posterior talofibular ligament.

194
Q

Performing a wall-sit (maintaining a squat stance against a wall) is an example of what type of exercise?

A Isometric
B Isoenergetic
C Isotonic
D Isokinetic

A

Answer: A

Explanation:
• In isometric exercise, NO movement occurs throughout the entire exercise.
• In isotonic exercise, the weight (tone) being applied to a muscle or joint is the same throughout the entire exercise (e.g. biceps curls - the dumbbell weighs the same, and, thus, imparts the same forces onto the arm for the entire exercise).
• In isokinetic exercise, the movement and rhythm of the exercise is the same throughout the entire exercise (e.g. cardiovascular machines such as the elliptical, cycling, rowing).

195
Q

A 29 year-old male presents to your clinic with complaints of 3 months of gradual onset right calf pain. The pain is worse during his basketball games. He also notes associated tingling in his toes. His symptoms are relieved by rest. He denies weakness. It is starting to affect his performance, however, so he asks you for help. On exam, tibial squeeze test is negative. Dorsalis pedis pulse is palpable during passive plantarflexion. What is the most appropriate next step?

A X-rays
B MRI
C Arteriogram
D Manometry testing

A

Answer: D

Explanation:
• Chronic exertional compartment syndrome (CECS): a chronic condition of temporarily raised intracompartmental pressures during exercise, causing muscle and nerve compression leading to pain and paresthesias
• Symptoms are relieved by rest because the reduced blood flow during rest lowers the amount of blood being pumped into the compartment, thus lowering the amount of pressure within the compartment.
• Workup involves compartment pressure testing (manometry) before and and immediately after exercise (best performed while the patient is experiencing their symptoms) to demonstrate pressure increase within the compartment of suspicion.
• Treatment for CECS is surgical fasciotomy.
• X-rays to rule out shin splints is a good idea, but less likely in this patient with a negative tibial squeeze test. Shin splints would also NOT cause paresthesias.
• Arteriogram is useful for diagnosing popliteal artery entrapment syndrome, but this is rare and less likely, given the palpable dorsalis pedis pulse during plantarflexion.
• MRI would NOT be indicated here, and certainly NOT as a first step over X-rays in this case.

196
Q

The second extensor compartment of the wrist contains which of the following tendons?

A Abductor pollicis brevis, extensor pollicis longus
B Extensor pollicis longus
C Abductor pollicis longus, extensor pollicis brevis
D Extensor carpi radialis longus, extensor carpi radialis brevis

A

Answer: D

Explanation:
• The second extensor compartment contains the Extensor carpi radialis longus (ECRL) and extensor carpi radialis brevis (ECRB) tendons.

197
Q

An 80 year-old female with history of stroke (minimal residual deficits) presents to your clinic with complaints of right groin pain. She denies trauma. It has developed gradually over the past several months along with low back pain. She denies numbness or tingling, but feels weak in her right leg. On exam, strength is neurologically intact. There is no tenderness to palpation of the anterior, lateral, or posterior right hip. Log roll reproduces her right groin pain. You have records demonstrating that she has completed a full course of physical therapy and has received a corticosteroid injection into the hip, both with only minimal relief. X-rays reveal significant reactive sclerosis, subchondral cysts, and asymmetric joint space narrowing of the femoroacetabular joint. What is the next best step?

A Orthopedic surgery consult
B EMG
C Prolotherapy injection
D PRP injection

A

Answer: A

Explanation:
• This patient with clear hip OA has failed conservative treatments as well as hip injection, and should be considered for hip replacement.
• Minimal relief with a hip injection does NOT offer a good prognosis for hip prolotherapy or PRP.

198
Q

Which of the following lab results would you expect to see in classic Sjogren syndrome? ANA: antinuclear antibody. RF: rheumatoid factor.

A ANA-, RF-
B ANA-, RF+
C ANA+, RF-
D ANA+, RF+

A

Answer: D

Explanation:
• Sjogren syndrome classically is ANA+ and RF+.

199
Q

The sixth extensor compartment of the wrist contains which of the following tendons?

A Extensor digiti minimi
B Extensor digitorum
C Extensor pollicis longus
D Extensor carpi ulnaris

A

Answer: D

Explanation:
• The sixth extensor compartment contains the ECU tendon.

200
Q

You are performing a physical exam on a patient with knee pain. You lie the patient prone, flex their knee 90 degrees, and apply an axial force upon the patient’s heel (force directed superiorly from the heel into the calcaneus and tibia). What is the purpose of this test?

A To diagnose MCL or LCL tears
B To diagnose ACL tears
C To diagnose OA
D To diagnose meniscus tears

A

Answer: D

Explanation:
• Apley Grind test is described here, and is a test for medial or lateral meniscus tears in the knee.

201
Q

A 44 year-old female is playing with her daughter, running down her staircase, when she jumps over the bottom few steps and lands on the ground, attempting to pivot and run to the left. Upon landing she notes sudden-onset left knee pain. In your office 1 week later she mentions that she has had gradual onset of swelling in the left knee since the event. She feels like the knee “catches” when she tries to bend it. She is able to ambulate on her own power. What physical exam maneuver would most likely provide you with the information you need to confirm the clinical diagnosis?

A Anterior drawer
B Lachman
C Posterior drawer
D McMurray

A

Answer: D

Explanation:
• Knee meniscus tear typically happens with a twisting motion under force (landing from a jump and pivoting, deep squatting and twisting) and the symptoms include popping sensation with knee pain and gradual swelling over the next 24 hours, along with clicking/catching/locking of the knee upon ROM.
• Physical exam tests for the meniscus integrity are the McMurray, Apley grind, Thessaly, deep squatting and twisting, and bounce home tests.
• Anterior drawer and Lachman tests are for diagnosing ACL tears, which is NOT a suspicious diagnosis in this patient.
• Posterior drawer test is for PCL tears, which typically occur in a dashboard motor vehicle accident injury.

202
Q

A 99 year-old male in impressive physical shape is completing an ironman competition. He sustains a mechanical fall on some loose rocks, causing a tear of the ulnar collateral ligament of the MCP the 1st digit. This is known by what name, and what is the most appropriate initial treatment?

A Skier’s thumb; thumb spica splint
B Skier’s thumb; surgery
C Stener’s lesion; thumb spica splint
D Stener’s lesion; surgery

A

Answer: A

Explanation:
• Skier’s thumb is a tear of the ulnar collateral ligament of the thumb MCP due to excessive radial deviation of the digit.
○ It is typically treated with thumb spica splinting.
• Stener’s lesion is a feared complication of Skier’s thumb by which the thumb adductor aponeurosis becomes trapped in the thumb MCP joint, due to the torn ligament now opening up access to the joint and allowing the aponeurosis to become trapped there.
Stener’s lesion requires orthopedic surgery consult.

203
Q

During a diagnostic ultrasound scan, you notice a structure that appears anechoic. Which of the following are you most likely viewing?

A Bone cortex
B Tendon
C Cyst
D Nerve

A

Answer: C

Explanation:
• On ultrasound, hyperechoic means bright white, hypoechoic means darker, and anechoic means black.
• If a structure is hyperechoic, it is bright because essentially all the sound waves hitting that structure are bouncing off it, right back to the transducer, where the computer interprets this intense signal by brightening it on the screen.
• Bone, thus, is hyperechoic, as no sound waves are penetrating through the cortex.
• Tendons and nerves can be hyper- or hypoechoic depending on how the transducer is angled upon them.
• Fluid, however, such as blood vessel lumens, cysts, bursae, is typically very hypoechoic or anechoic, as the sound waves travel right through the fluid instead of bouncing back at the transducer.
Thus, the computer is NOT receiving any sound waves bouncing back from the fluid, and colors that region of the screen dark as a result.

204
Q

The concept of increasing the weight of a resistance exercise over weeks and months in order to build strength is known as:

A Getting huge
B Muscle-building
C Progressive resistance
D Strength exercise

A

Answer: C

Explanation:
This is known as progressive resistance exercise, also known as progressive overload.

205
Q

A 30 year-old male is involved in a bicycle accident. He presents to the ED with right forearm pain. On exam he demonstrates intact sensation, but impaired strength secondary to severe pain. Trauma x-rays reveal a distal radius fracture in which the distal radius fragment is volarly displaced. This type of fracture is known by which of the following names?

A Monteggia
B Galeazzi
C Smith
D Colles

A

Answer: C

Explanation:
• A distal radius fracture is typically caused by trauma or fall on outstretched hand (FOOSH).
• Distal radius fractures happen in two ways: the distal radius fragment is dorsally displaced (Colles fracture), or volarly displaced (Smith fracture).
Both require orthopedic surgery consultation.

206
Q

A 42 year-old male presents with gradual onset bilateral foot pain. He denies trauma, numbness, or tingling. His pain is worse in the morning and improves throughout the day. You note pes planus on exam, as well as reproduction of pain with great toe extension while palpating the proximal medial sole of the foot. What is the most appropriate next step?

A Orthopedic surgery consultation
B Corticosteroid injection
C Reassurance
D Physical therapy

A

Answer: D

Explanation:
• Plantar fasciitis (tightness and inflammation of the plantar fascia): gradual onset foot pain worse in the morning, improving throughout the day
• Risk factors include pes cavus, pes planus, and tight achilles tendons.
• Physical exam is sufficient to diagnose plantar fasciitis; the classic maneuver involves extending the great toe while palpating the medial plantar surface just distal to the calcaneus (the origin of the plantar fascia off the calcaneus). If this reproduces the patient’s pain, then they most likely have plantar fasciitis.
• Treatment involves comfortable orthotics, nighttime dorsiflexion splints, physical therapy for plantar fascia stretching, NSAIDs, and surgical release in chronic, resistant cases.
Corticosteroid injection into/near the plantar fascia may cause plantar fascia rupture and should generally be avoided.

207
Q

A 48 year-old male presents to your clinic with the complaint of elbow problems. He notes that for the past several weeks he has noticed his right elbow is “bigger” than the left elbow. He is a visual artist. On exam, strength and sensation are intact. You notice that at the posterior elbow there is a very large swelling. It is compressible and warm. The patient denies pain. He asks if this is serious. What is the most likely diagnosis?

A Septic arthritis
B Anterior elbow dislocation
C Posterior elbow dislocation
D Olecranon bursitis

A

Answer: D

Explanation:
• Olecranon bursitis is usually due to repetitive forces to the olecranon, such as grinding your elbow on a desk for several hours daily (art, desk work, etc.).
• A large pouch of fluid develops at the olecranon bursa because there is very little skin there to prevent it from expanding if it is inflamed.
• Treatment involves RICE, elbow pad, and considering aspiration.
• It is not usually an infectious process, but if the patient has a fever, then consider aspiration, culture, and antibiotics.
• Elbow dislocations would occur with a history of trauma, and would be painful. Septic arthritis would be accompanied by fever and joint pain.

208
Q

A 48 year-old male presents to your clinic with the complaint of elbow problems. He notes that for the past several weeks he has noticed his right elbow is “bigger” than the left elbow. He is a visual artist. On exam, strength and sensation are intact. You notice that at the posterior elbow there is a very large swelling. It is compressible and warm. The patient denies pain. He asks if this is serious. What is the most likely diagnosis?

A Septic arthritis
B Anterior elbow dislocation
C Posterior elbow dislocation
D Olecranon bursitis

A

Answer: D

Explanation:
• Olecranon bursitis is usually due to repetitive forces to the olecranon, such as grinding your elbow on a desk for several hours daily (art, desk work, etc.).
• A large pouch of fluid develops at the olecranon bursa because there is very little skin there to prevent it from expanding if it is inflamed.
• Treatment involves RICE, elbow pad, and considering aspiration.
• It is not usually an infectious process, but if the patient has a fever, then consider aspiration, culture, and antibiotics.
• Elbow dislocations would occur with a history of trauma, and would be painful. Septic arthritis would be accompanied by fever and joint pain.

209
Q

Which of the following is an example of an isokinetic exercise?

A Leg press
B Planks
C Elliptical machine
D Biceps curls

A

Answer: C

Explanation:
• In isometric exercise, NO movement occurs throughout the entire exercise.
• In isotonic exercise, the weight (tone) being applied to a muscle or joint is the same throughout the entire exercise (e.g. biceps curls - the dumbbell weighs the same, and, thus, imparts the same forces onto the arm for the entire exercise).
• In isokinetic exercise, the movement and rhythm of the exercise is the same throughout the entire exercise (e.g. cardiovascular machines such as the elliptical, cycling, rowing).

210
Q

Which of the following is the function of the iliofemoral ligament regarding the hip?

A Limit abduction, flexion, internal rotation
B Limit abduction, extension, internal rotation
C Limit abduction, extension, external rotation
D Limit abduction, flexion, external rotation

A

Answer: C

Explanation:
• Iliofemoral ligament extends from the ileum to femur on the anterior side of the hip joint. Its function is to limit hip abduction, extension, and external rotation.

211
Q

A 23 year-old male twists his ankle in a football game when a lineman is pushed backwards into him. You rush onto the field and diagnose a torn tibiofibular interosseous ligament. Which of the following is an important, common complication of this injury that you must rule out?

A Distal fibula fracture
B Proximal fibula fracture
C Proximal tibia fracture
D Distal tibia fracture

A

Answer: B

Explanation:
• Maisonneuve fracture is a proximal fibula fracture that may occur following a high ankle sprain (tibiofibular syndesmosis tear/rupture).
• Essentially the tearing of the syndesmosis extends all the way up proximally until the forces continue into the proximal fibula, causing a fracture there.
• X-rays of the knee are thus required following a high ankle sprain.
• Maisonneuve fracture generally requires orthopedic surgery consultation.

212
Q

A 27 year-old female falls on her outstretched hand and develops sudden-onset hand pain. She is taken to the ED where x-rays reveal a scaphoid fracture in the distal ⅓ of the bone. What is the next best step?

A CT scan
B Orthopedic surgery consult
C MRI
D Thumb spica cast

A

Answer: D

Explanation:
• Scaphoid fractures present following trauma/FOOSH (fall on outstretched hand) and typically cause pain in the anatomic snuffbox.
• X-rays/CT scan/MRI may reveal a fracture of the scaphoid.
• If this fracture occurs within the proximal ⅓ of the bone, orthopedic surgery consultation is required.
• If within the distal ⅔ of the bone, thumb spica cast is appropriate.
• If initial X-rays are negative for fracture, but you still suspect a scaphoid fracture, place thumb in thumb spica cast and repeat X-rays in 2 weeks, as the fracture may reveal itself at that time.
• You can also consider CT/MRI if X-rays are negative.
• Pain in the anatomic snuffbox is a scaphoid fracture until proven otherwise, even if initial X-rays are negative.

213
Q

A 56 year-old male presents to your clinic with shoulder pain for the past 3 months. He went to the ED initially when it began, and shoulder xrays were negative. He denies history of trauma. On exam you note positive O’Brien, negative empty can, negative Neer, and negative Hawkins. Strength is intact. There is no numbness or tingling. He has tried several weeks of physical therapy with minimal benefit. What is the next best step?

A Shoulder MR arthrogram
B Shoulder MRI
C Shoulder CT scan
D Repeat shoulder xrays

A

Answer: A

Explanation:
• Shoulder pain and positive O’Brien test (a test for labral tears) presents with history and exam suspicious for labrum tear.
• This is best evaluated on MRI with contrast injected into the shoulder (known as MRI arthrogram).
• If a labral tear exists, the injected dye will flow into the labral defect, “lighting it up” on MRI.
• X-rays do NOT show soft tissue pathology well, and CT/regular MRI will NOT show labral tear nearly as well as MR arthrogram.

214
Q

Which of the following muscles is NOT a knee internal rotator?

A Gracilis
B Sartorius
C Semitendinosus
D Biceps femoris

A

Answer: D

Explanation:
• Biceps femoris is a knee external rotator.
• The remaining three muscles are part of the “Say Grace before Tea” mnemonic (muscles that attach to the pes anserine, located on the medial tibia), rendering them all knee internal rotators.

215
Q

You are reviewing left lower limb EMG/NCS findings and trying to propose a possible treatment plan to the patient. The patient is experiencing foot pain not relieved by NSAIDs or heat/ice. NCS reveals normal superficial fibular and sural SNAPs (sensory nerve action potentials), prolonged latency of medial and lateral plantar nerve SNAPs, and abnormal CMAP (compound muscle action potential) to abductor hallucis (AH). The CMAP to the extensor digitorum brevis (EDB) is normal. Needle EMG reveals decreased recruitment in AH, abductor digiti quinti pedis (ADQP), and lumbricals. EMG of tibialis anterior, EDB, gastrocnemius, fibularis longus, hamstrings, rectus femoris, tensor fascia lata, and lumbosacral paraspinals is normal. Which of the following is the most reasonable treatment strategy for this condition?

A Peripheral nerve stimulation
B Gabapentin trial
C Surgical referral
D Bracing

A

Answer: C

Explanation:
• Tarsal tunnel syndrome, a rare compression of the tibial nerve as it passes through the tarsal tunnel around the medial malleolus, underneath the flexor retinaculum
• Flexor retinaculum is a common culprit that compresses the tibial nerve too tightly, causing the patient’s symptoms of plantar foot pain with numbness/tingling and possible foot muscle weakness.
• Surgical release of the flexor retinaculum (akin to carpal tunnel release) is the best option of these choices (a structural solution for a structural problem).
• US-guided tarsal tunnel injection could also be considered. Bracing would not do anything to decompress the tarsal tunnel. Gabapentin also would not decompress the tarsal tunnel, but mask the symptoms while the nerve compression continues, potentially destroying axons over time. Peripheral nerve stimulation is not appropriate for this reason also.

216
Q

A grade 2 lateral ankle sprain is defined as which of the following? ATFL: anterior talofibular ligament. CFL: calcaneofibular ligament. PTFL: posterior talofibular ligament.

A Fully torn ATFL, partially torn PTFL
B Fully torn ATFL, partially torn CFL
C Fully torn ATFL, fully torn CFL
D Partially torn ATFL, intact CFL

A

Answer: B

Explanation:
• Grade 1 lateral ankle sprain: partially torn ATFL, intact CFL.
• Grade 2: fully torn ATFL, partially torn CFL.
• Grade 3: fully torn ATFL and CFL.

217
Q

The PCL (posterior cruciate ligament of the knee) becomes loose with which of the following movements?

A Knee flexion
B Knee extension
C Knee internal rotation
D Knee external rotation

A

Answer: B

Explanation:
• The PCL runs posteroinferolaterally from the femur toward the tibia.
• It tenses with knee flebxion. It limits translation of the tibia in the posterior direction.

218
Q

With aerobic training, a patient can alter the ratio of which of the following?

A Type IIb to Type IIa muscle fibers (increased Type IIb vs. Type IIa)
B Type IIa to Type IIb muscle fibers (increased Type IIa vs. Type IIb)
C Type I to Type IIb muscle fibers
D Type I to Type IIa muscle fibers

A

Answer: B

Explanation:
• With aerobic training, a patient can increase their ratio of type IIa to type IIb muscle fibers (increasing the proportion of type IIa fibers relative to type IIb).
• Type I muscle fiber amount remains roughly the same.

219
Q

An example of a closed chain exercise would be:

A Biceps curls
B Leg extensions
C Bench press
D Push-ups

A

Answer: D

Explanation:
• Closed chain exercises involve the distal segment of the limb being fixed (not moving).
• Open chain exercises involve the distal segment of the limb being NOT fixed (moving in space).
• Of all the exercises listed, only push-ups have the distal arm (the hand) being fixed against the ground and not moving.

220
Q

A 27 year-old female presents with complaints of right foot tingling and pain. She is a marathon runner. She denies weakness. On exam, straight leg raise is negative. You decide to grab the foot and squeeze the bones together in the horizontal plane. This reproduces her foot pain with tingling. What is the next most appropriate step?

A Metatarsal pads
B Surgical consultation
C Foot MRI
D Corticosteroid injection

A

Answer: A

Explanation:
• Morton neuroma, a benign growth of the interdigital nerve between two metatarsals (usually the 3rd and 4th intermetatarsal space).
• This can be due to repetitive microtrauma to that nerve, e.g. during marathon training or other sports.
• The Morton click is the physical exam maneuver described; by squeezing the problematic metatarsal heads together and reproducing pain, dysesthesias/paresthesias, and typically a clicking sensation, you demonstrate this positive physical exam finding for Morton neuroma.
• Treatment involves metatarsal pads, comfortable orthotics, wide toe box shoes, US-guided corticosteroid injection, and the last resort of surgery.
• Foot X-rays can be useful to rule out other causes of pain, such as a stress fracture, and would be indicated prior to ordering an MRI.
• Foot MRI would be useful in cases of a negative foot X-ray and lack of response to conservative treatments +/- corticosteroid injection.

221
Q

With LCL (lateral collateral ligament) injuries in the knee, it is important to rule out nerve injury most crucially in which of the following nerves?

A Popliteal nerve
B Tibial nerve
C Common fibular nerve
D Sciatic nerve

A

Answer: C

Explanation:
• Common fibular nerve wraps around the fibular head/neck near the LCL attachment, so with injuries to the lateral collateral ligament (LCL) come injuries to other nearby structures, e.g. the common fibular nerve.
• Thus, it is important to rule out injury to this nerve in a patient with LCL tear.
• High-yield functions to test would be foot dorsiflexion and eversion.

222
Q

A 54 year-old female with history of systemic lupus erythematosus (SLE) presents with gradual onset left groin pain. She denies history of trauma, weakness, numbness, or tingling. You choose to order x-rays because you are suspicious for which of the following diseases?

A Proximal femoral stress fracture
B Slipped capital femoral epiphysis
C Intracapsular femur fracture
D Avascular necrosis of the femoral head

A

Answer: D

Explanation:
• Avasclular necrosis (AVN) of the femoral head occurs with gradual onset groin pain secondary to poor, severely decreased blood flow to the femoral head, frequently due to congenital causes (Legg-Calve-Perthes disease), chronic corticosteroid usage, alcohol, and SLE (lupus).
○ This patient has lupus and is at risk for chronic corticosteroid usage as well because of this.
○ AVN of the femoral head requires orthopedic surgery consult.
• Proximal femoral stress fracture typically occurs in underweight athletes training for long distance-running with other risk factors such as amenorrhea, disordered eating, osteoporosis.
• Slipped capital femoral epiphysis (SCFE) occurs typically in obese adolescent males.
• Intracapsular femur fracture typically occurs with trauma.

223
Q

A 31 year-old female presents with elbow pain. On examination you elicit elbow pain while palpating the common extensor tendon during resisted wrist extension. This maneuver is known as:

A D’Angelo test
B Cozen test
C Maudsley’s test
D Mill’s test

A

Answer: B

Explanation:
• Mill’s test is keeping the elbow extended while the physician flexes the wrist and pronates the forearm, reproducing lateral epicondylitis pain.
• Maudsley’s test involves resisted 3rd finger extension while palpating the lateral epicondyle, reproducing lateral epicondylitis pain.

224
Q

A patient develops gradual onset knee pain due to a redundant fold of synovial tissue in the knee that has become thickened and inflamed, leading to knee pain with locking and catching of the knee. This is known as what condition?

A Congenital fibrous extrusion syndrome
B IT band syndrome
C Plica syndrome
D Patellofemoral pain syndrome

A

Answer: C

Explanation:
• Knee plica causing pain and inflammation
• It is best treated with physical therapy, corticosteroid injection into the plica, or surgical resection of the plica.

225
Q

A positive geyser sign at the acromioclavicular (AC) joint most likely indicates which of the following pathologies?

A None of these answers is correct
B Osteoarthritis
C Rotator cuff tendonitis
D Labral tear

A

Answer: B

Explanation:
• Geyser sign refers to the concept of viewing the AC joint on ultrasound and detecting a large amount of fluid within the joint which pushes upward against the AC ligament, pushing it upward and outward, much like a geyser of fluid.
• Osteoarthritis in the AC joint is most likely to cause this.

226
Q

A 19-year old long distance runner is training for a marathon when he develops gradual onset bilateral medial shin pain. He denies trauma, weakness, numbness, or tingling. On exam, strength is intact. Based on this limited information, what is the most likely diagnosis?

A Patellofemoral pain syndrome
B Chronic exertional compartment syndrome
C Medial tibial stress syndrome
D Tibial stress fracture

A

Answer: C

Explanation:
• Bilateral shin pain that occurs in the context of increased long distance running without trauma is highly indicative of shin splints (medial tibial stress syndrome).
• With shin splints, excessive overload on the tibia causes microtears at the muscle-periosteum interface, essentially chronically pulling on the periosteum, which, quite simply, hurts.
• Stress fractures would be another reasonable concern, but if you must pick between the two options and the clinical example is relatively simple, pick shin splints.

227
Q

A 64 year-old female presents to your clinic with complaints of 2 months of sciatica. She notes burning left buttock pain radiating down her posterior left thigh, stopping at the knee. Her pain is often relieved with shifting her weight onto her right buttock when sitting. She notes some tingling in her left buttock. She denies bowel or bladder dysfunction, and denies saddle anesthesia. On exam she has 5/5 strength in the left lower limb, intact sensation, and normal reflexes. Left slump sit causes left posterior thigh pain. There is tenderness to palpation of the left proximal medial buttock. There is left buttock pain with resisted knee flexion. What is the most likely diagnosis?

A Piriformis syndrome
B Lumbosacral radiculitis
C SI joint dysfunction
D Ischial bursitis

A

Answer: D

Explanation:
• Ischial bursitis
• Ischial bursa lies between the ischial tuberosity (your “sit” bone on both sides) and the gluteus maximus muscle.
• It can be inflamed in some patients, causing pain, worse with palpation of the ischial bursa, and worse with sitting (relieved with shifting weight onto the normal buttock, thus relieving pressure off the inflamed bursa).
• The ischial tuberosity is also the origin of the hamstrings, and realistically hamstring strains or tendonitis is more likely than ischial bursopathy, but both present very similarly to this patient.
• Treatment of ischial bursitis involves rest, ice, NSAIDs, PT, corticosteroid injection into the bursa.
• Piriformis syndrome would cause lateral buttock pain closer towards the greater trochanter. Greater trochanteric pain would be lateral on the proximal thigh.
• SI joint dysfunction would cause a low back pain with positive SI joint tests.

228
Q

A 32 year-old male with a known history of right femoroacetabular impingement (FAI) presents to you with 2 months of worsening right hip pain associated with a clicking sensation in the groin. He previously performed physical therapy and had x-rays taken of his right hip, which demonstrated FAI. Physical therapy improved his symptoms and you discharged him from your care 6 months ago; he continues his home exercise program. He denies trauma. He plays catcher for his local softball team. On exam, log roll, hip scour, and anterior hip impingement test are positive. What is the next best step?

A Orthopedic surgery consultation
B Repeat x-rays
C MR arthrogram
D MRI

A

Answer: C

Explanation:
• Femoroacetabular impingement (FAI) is at risk for hip labrum tear, which is demonstrated here.
• It can be associated with clicking/snapping inside the groin, and is at increased risk of occurrence in patients with FAI.
• This patient without trauma is NOT suspicious for hip fracture. Repeat X-rays would likely not add any additional information.
• MR arthrogram is the imaging test of choice in cases of suspected labral tear, as the dye will flow into the labral defect, which is not a feature possible in conventional MRI.

229
Q

A 59 year-old male is seen in follow-up for stroke resulting in left hemiparesis. He notes left shoulder pain since his stroke. On examination, he is sensitive to touch over his left deltoid. Speed, Hawkins, and Neer tests are positive. On x-ray of the left shoulder, you note a high-riding humeral head. What is the most likely diagnosis?

A Shoulder dislocation
B Biceps tendonitis
C Complex regional pain syndrome type 1 (CRPS 1)
D Rotator cuff tear (supraspinatus)

A

Answer: D

Explanation:
• Patients with stroke commonly develop “bread and butter” musculoskeletal problems just like patients without stroke.
• This patient presents with history and exam findings suggestive of rotator cuff tear.
• The X-ray findings of a superiorly migrated humerus indicate a torn supraspinatus, whose fibers have retracted and NO longer occupy the subacromial space which the humerus has now elevated into.
• None of the other pathologies will demonstrate this X-ray finding.
• Physical exam findings are often pan-positive in patients with severe pain.

230
Q

A 29 year-old female presents to your clinic with complaints of 1 month of left wrist pain. Her pain is intermittent and mild. She gave birth to her new baby daughter 6 weeks ago. She denies numbness or tingling. On exam, strength is intact. When tucking the thumb into the palm and making a fist, then ulnarly deviating the wrist, the patient experiences sudden-onset exquisite pain that reproduces her usual wrist pain. What is the most appropriate next step?

A Ultrasound-guided injection
B Reassurance
C Orthopedic surgery consult
D NSAIDs, thumb spica splint

A

Answer: D

Explanation:
• This patient with a positive Finkelstein test as noted in the question stem, has findings highly suspicious for 1st extensor compartment tenosynovitis, or De Quervain Tenosynovitis (inflammation of the abductor pollicis longus and/or extensor pollicis brevis tendon sheath).
• This commonly occurs in new mothers who are spending a lot of time straining their wrists, holding their new baby.
• If conservative measures have failed, such as oral over the counter medications, and/or the pain is constant and severe, then ultrasound-guided corticosteroid injection into the tendon sheath (not the tendon itself!) is indicated, and is highly effective for this disease.
• However, in this patient, her pain is only mild, and she is NOT yet attempted conservative measures such as NSAIDs or thumb spica splinting; these should be tried first.

231
Q

A 19 year-old ballet dancer presents with a painful snapping/clicking sensation in her right groin, worse while she dances. She has not had any treatment or imaging for this. She denies weakness, numbness, or tingling. What is the most likely mechanism of the problem she is experiencing?

A Subluxation of the IT band over the greater trochanter
B Subluxation of the iliopsoas tendon over the iliopectineal eminence
C Subluxation of the sartorius tendon over the ASIS
D Subluxation of the rectus femoris tendon over the AIIS

A

Answer: B

Explanation:
• Internal snapping hip synbdrome is groin pain caused by the iliopsoas tendon snapping/subluxing over the iliopectineal eminence.
○ It can be associated with iliopsoas tendonitis/bursitis.
○ If a patient complains of a snapping sensation in the groin, they are likely correct: something is snapping, and the location determines what is snapping.
○ If in the groin, then it is likely internal snapping hip syndrome as discussed here.
• If snapping occurs over the proximal lateral thigh, then it is likely external snapping hip syndrome, which is caused by a tight IT band snapping over the greater trochanter.
• Treatment for both involves RICE, NSAIDs, PT, corticosteroid injection.

232
Q

A 34 year-old male falls on his outstretched hand. X-rays reveal a fracture of the scaphoid bone occurring in the proximal ⅕ of the bone. What is the most appropriate next step?

A Orthopedic surgery consult
B Thumb spica cast, repeat x-rays in 2 weeks
C MRI
D CT scan

A

Answer: A

Explanation:
• Pain in the anatomic snuffbox following trauma
• Scaphoid fractures present following trauma/FOOSH (fall on outstretched hand) and typically cause pain in the anatomic snuffbox.
• X-rays/CT scan/MRI may reveal a fracture of the scaphoid.
• If this fracture occurs within the proximal ⅓ of the bone, orthopedic surgery consultation is required.
• If within the distal ⅔ of the bone, thumb spica cast is appropriate.
• If initial X-rays are negative for fracture, but you still suspect a scaphoid fracture, place thumb in thumb spica cast and repeat X-rays in 2 weeks, as the fracture may reveal itself at that time.
• You can also consider CT/MRI if X-rays are negative.
• Pain in the anatomic snuffbox is a scaphoid fracture until proven otherwise, even if initial X-rays are negative.

233
Q

A 27 year-old male is running a marathon when he trips and lands on his right shoulder. He experiences immediate severe shoulder pain. On exam, there is asymmetry when comparing his right shoulder to his left. You diagnose an anterior shoulder dislocation. This patient’s ultimate treatment will most likely be which of the following?

A None of these answers is correct
B Surgery
C Rehab alone
D Reassurance

A

Answer: B

Explanation:
• Patients with traumatic unilateral shoulder dislocations typically require surgery.

234
Q

A 14 year-old male with a history of obesity presents with sudden-onset left hip pain. He denies trauma. He has been healthy other than a diarrheal illness 2 weeks ago. Hip x-rays demonstrate a widened growth plate and femoral head sliding off the growth plate by approximately 40%. Which of the following grades could correctly be assigned to these x-ray findings?

A 4
B 3
C 2
D 1

A

Answer: C

Explanation:
	• Slipped capital femoral epiphysis (SCFE) involves sudden-onset hip/groin pain due to the femoral head sliding off the growth plate, typically in obese adolescent males.
	• X-ray grading is defined by the percentage of slippage of the femoral head off the growth plate.
	• Grade 1: 0 - 33% sliding.
	• Grade 2: 34 - 50% sliding.
	• Grade 3: > 50% sliding.
	• There is NO grade 4.
	• Treatment is orthopedic surgery.
235
Q

In which of the following cases is microwave diathermy most appropriate?

A Therapeutic application during pregnancy
B To hasten the breakdown of a hematoma
C Heating superficial tendons
D Heating deep tendons

A

Answer: B

Explanation:
• Ultrasound uses ~1 MHz sound waves to produce vibrations below the skin and produce images out of those vibrations and sound wave reflections.
○ It is most useful to heat deep tendons, ligaments, and for general MSK pain (up to 8cm below the skin).
○ Ultrasound is NOT appropriate near electrical implants or cancer.
• Short wave diathermy uses ~27 MHz radio waves to heat up tissues up to 5cm deep.
• Microwave diathermy uses ~1000 MHz microwaves to heat very superficial structures 1-3 cm deep, and is indicated to hasten the breakdown of hematomas.
○ Diathermy is contraindicated over metal and cancer especially.

236
Q

A 37 year-old powerlifter is performing a 1,300lb deadlift in a competition, left arm pronated and right arm supinated, when he notices sudden-onset right anterior elbow pain, swelling, and bruising. He cries out, “doc, you’ve got to help me!”. Hook test is positive. X-rays reveal an avulsion fragment off the radial tuberosity. What is the most appropriate next step?

A Surgery
B Corticosteroid injection
C PT
D RICE

A

Answer: A

Explanation:
• This patient with ruptured distal biceps tendon and avulsion fracture on X-ray requires orthopedic surgery.

237
Q

In a patient with bilateral knee and finger pain, symmetric joint space narrowing, and positive anti-CCP antibodies, which of the following synovial fluid study results is the most likely?

A Increased neutrophils; 150k WBCs
B Decreased neutrophils; 150k WBCs
C Increased neutrophils; 75k WBCs
D Decreased neutrophils; 75k WBCs

A

Answer: C

Explanation:
• Rheumatoid arthritis: synovial fluid analysis shows increased neutrophils (approximately 75% predominance) with elevated, but under 100k WBC.
• Septic arthritis: the joint fluid may show >75% neutrophils with > 100k WBCs.

238
Q

A patient presents to your clinic with 5 months of right shoulder pain. On exam, you attempt passive abduction of the shoulder, which takes greater than usual force for you to abduct it, and the patient cries out in pain when you reach 70 degrees of abduction. Empty can, Neer, Hawkins, Speed, and O’Brien tests are positive. What is the most likely diagnosis?

A Rotator cuff tear
B Frozen shoulder
C Subacromial impingement/bursitis
D Labral tear

A

Answer: B

Explanation:
• The key to answering this is the high difficulty in simply abducting the shoulder, as well as the limited degrees of abduction, which together imply a very tight shoulder capsule.
• Frozen shoulder (adhesive capsulitis) frequently shows pan-positive exam findings suggestive of other diseases.
• The other answer choices by themselves would not restrict shoulder range of motion to the same degree that frozen shoulder does.

239
Q

A 58 year-old female complains of sudden-onset severe shoulder pain for 2 weeks, which then improves and is followed by progressive onset shoulder weakness. She denies trauma. Shoulder x-ray and MRI are negative. On exam she has weakness in shoulder abduction. EMG/NCS reveals decreased recruitment and +2 positive sharp waves in the supraspinatus and infraspinatus. Which of the following is the most reasonable advice you should give to this patient?

A All of the listed answer choices are equally reasonable
B Repeat EMG within 6 months is not typically useful
C Nerve transposition should be considered early in the course of the disease
D Most cases self-resolve within 1-2 years

A

Answer: D

Explanation:
• Parsonage-Turner Syndrome (PTS, neuralgic amyotrophy, idiopathic brachial plexopathy)
• This typically presents as sudden shoulder pain for 2 weeks which gives way to weakness, classically in the suprascapular (this case), long thoracic, and/or anterior interosseous nerve territory.
• Serial EMGs are useful for prognosis, and most cases self-resolve within 1-2 years after onset.
• Early surgical options would NOT be indicated, as most cases self-resolve.
• PTS typically begins following a viral illness or surgery, and its etiology is NOT entirely defined at this time.

240
Q

A 36 year-old male professional wrestler twists his knee and suffers a lateral meniscus tear. MRI reveals a large tear of the outer 1/5 of the meniscus. What is the most appropriate treatment option?

A Surgical repair of the meniscus
B Surgical resection of the meniscus
C Knee bracing
D Physical therapy

A

Answer: A

Explanation:
• Tears of the inner 2/3 of the knee meniscus are appropriate for surgical resection (removal), as the inner 2/3 of the meniscus has poor blood supply, rendering surgical repair ineffective.
• However, the outer 1/3 has good blood supply; thus, surgical repair is appropriate for injuries to the outer 1/3 of the meniscus.

241
Q

The first extensor compartment of the wrist contains which of the following tendons?

A Abductor pollicis longus, extensor pollicis longus
B Abductor pollicis longus, extensor pollicis brevis
C Abductor pollicis brevis, extensor pollicis longus
D Abductor pollicis brevis, extensor pollicis brevis

A

Answer: B

Explanation:
• First extensor compartment contains the APL and EPB tendons.

242
Q

A 44 year-old male is involved in a bar fight. In the ED, x-rays reveal discontinuity of bone cortex at the 3rd metacarpal neck. This is commonly known as which of the following?

A Boxer’s fracture
B Mallet finger
C Rower’s fracture
D Colles fracture

A

Answer: A

Explanation:
• Metacarpal neck or shaft fracture is called a Boxer’s fracture.
• Treatment is either conservative or surgical, but orthopedic surgery consult is required.

243
Q

A 22 year-old male presents to your clinic with the chief complaint of right foot and ankle pain with intermittent swelling. The pain is worse with athletic activity, and improved with rest. There is tingling in the dorsum of the foot. On exam, the dorsalis pedis pulse diminishes when the foot is placed into plantarflexion. What is the most appropriate diagnostic test to perform?

A EMG
B MRI
C Arteriogram
D Compartment manometry

A

Answer: C

Explanation:
• Popliteal artery entrapment syndrome (PAES) involves the popliteal artery in the knee becoming compressed in the popliteal fossa due to an abnormal arterial course or knee muscles impinging upon it abnormally.
• It presents with lower limb swelling and pain with paresthesias due to nerve compression.
• The classic physical exam maneuver is to demonstrate decreased dorsalis pedis pulse when the ankle is plantarflexed (thus putting the gastrocnemius in a position to pinch the popliteal artery).
• Workup involves arteriogram to demonstrate abnormal arterial course or flow.

244
Q

A 24 year-old female sustains a fall onto both of her elbows while her shoulders are in a flexed, adducted position, and experiences immediate right shoulder pain as a result. What is her most likely diagnosis?

A None of these answers is correct
B AC joint separation
C Posterior shoulder dislocation
D Anterior shoulder dislocation

A

Answer: C

Explanation:
• A flexed, adducted arm position places the shoulder in prime position to be susceptible to a posterior dislocation.
• An abducted, externally rotated shoulder is typically how anterior shoulder dislocations occur, which, indeed, occur far more frequently than posterior dislocations.

245
Q

A 38 year-old male presents with gradual onset bilateral foot pain. He denies trauma, numbness, or tingling. His pain is worse in the morning and improves throughout the day. You note pes planus and tight achilles tendons on exam. What workup will you most likely require?

A Physical examination alone
B EMG
C MRI
D X-rays

A

Answer: A

Explanation:
• Plantar fasciitis (tightness and inflammation of the plantar fascia), classically described as gradual onset foot pain worse in the morning, improving throughout the day.
• Risk factors include pes cavus, pes planus, and tight achilles tendons.
• Physical exam is sufficient to diagnose plantar fasciitis; the classic maneuver involves extending the great toe while palpating the medial plantar surface just distal to the calcaneus (the origin of the plantar fascia off the calcaneus). If this reproduces the patient’s pain then they most likely have plantar fasciitis.
• Treatment involves comfortable orthotics, nighttime dorsiflexion splints, plantar fascia stretching, NSAIDs, and surgical release in chronic, resistant cases.
• In this case of no weakness, numbness, or tingling, EMG would not be appropriate. This is a clinical diagnosis, and further imaging or EMG is not required initially.

246
Q

Dystonia is defined as which of the following?

A Involuntary muscle contraction
B Velocity-independent muscle resistance to passive stretch
C Involuntary abnormal muscle contractions resulting in abnormal posturing
D Contralateral sternocleidomastoid (SCM) spasm resulting in abnormal posturing

A

Answer: C

Explanation:
• Dystonia is defined as abnormal muscle contractions, involuntary, that result in abnormal posturing.
• Involuntary muscle contraction alone is NOT enough to make the diagnosis.
• Velocity-independent muscle resistance to passive stretch is known as rigidity.
• Contralateral SCM spasm may be a component of torticollis, not dystonia as a whole.

247
Q

Which of the following injuries correctly describes a Jones fracture?

A Fracture across the head of the 1st metatarsal
B Fracture across the base of the 1st metatarsal
C Fracture across the head of the 5th metatarsal
D Fracture across the base of the 5th metatarsal

A

Answer: D

Explanation:
• Jones fracture is a fracture across the base of the 5th metatarsal.
• Treatment is non-weight-bearing vs. surgery depending on type and severity.

248
Q

A 36 year-old female presents to your clinic for the evaluation of right shoulder pain. It has been present for the past year, and is worse if she reaches overhead. She denies weakness, numbness, or tingling. On exam, strength is intact. There is a positive empty can and Hawkins testing. Which of the following is the next best step?

A Start physical therapy for the right shoulder
B Perform diagnostic ultrasound scan of right shoulder
C Order right shoulder xray
D Examine the scapulae

A

Answer: D

Explanation:
• Rotator cuff disease/impingement
• The therapy for this would correctly be physical therapy.
• However, scapular dyskinesis is a common cause of rotator cuff pathology.
• Recall that in shoulder abduction, there are 60 degrees of scapulothoracic motion and 120 degrees of glenohumeral motion.
• If the scapulae do NOT move properly, then the rotator cuff is susceptible to impingement.
• Thus, it is imperative to examine biscapular mobility to ensure symmetry and that there is NO abnormal scapular winging on exam.
• Otherwise you might miss the root cause of this patient’s rotator cuff disease, and the patient is likely to continue damaging her rotator cuff. Physical therapy directed at proper biscapular mobility in addition to the rotator cuff can treat this condition.

249
Q

A swollen distal interphalangeal joint is otherwise known as which of the following?

A Boutonniere deformity
B Swan neck deformity
C Heberden node
D Bouchard node

A

Answer: C

Explanation:
• Heberden node is a swollen DIP is typically due to osteoarthritis of the DIP joint
• Bouchard node is a swollen PIP (proximal interphalangeal joint) due to rheumatoid arthritis.
• Swan neck and Boutonniere deformities are found in rheumatoid arthritis patients with poorly controlled chronic disease.

250
Q

You are examining a patient with right knee pain. The patient is seated with the hip flexed and knee extended. You flex their knee beyond 90 degrees, and then let it drop into extension via gravity. What is the purpose of this test?

A To diagnose MCL tears
B To diagnose LCL tears
C To diagnose meniscus tears
D Apprehension test for instability

A

Answer: C

Explanation:
• Bounce Home physical exam maneuver is a test for medial or lateral meniscus tears in the knee.
• A positive test yields an inability for the knee to fully extend via gravity alone, due to a mechanical blockage in the knee (i.e. the meniscus tear itself).

251
Q

A 74 year-old female falls onto her outstretched hand and sustains a proximal humerus fracture. On xrays, you notice that the humerus is intact except for the greater tuberosity which is displaced. What is the next best step?

A Reassurance
B Orthopedic surgery referral
C Arm sling
D Pain control

A

Answer: B

Explanation:
• Proximal humerus fractures are classified using the 4-part classification system.
• In 1-part fractures, the humerus remains as one whole part (intact), with no elements displaced off the humerus. These are treated with a sling.
• In 2-part fractures, one of the 4 “parts” (greater and lesser tuberosities, humeral head, humeral shaft) is displaced from the other 3, thus leaving 2 “parts” of bone existing in the proximal arm: the displaced fragment and the still intact other 3 parts.
• In 3-part fractures, two of these parts are broken off
• In 4-part fractures, all 4 of these parts are separated from one another.
• 2-4 part fractures require surgery, as in this question’s case.

252
Q

A 19 year-old male presents to your MSK clinic with complaints of gradual onset right elbow pain. He says he has been working out for the past month to “get huge”, and his favorite weight training exercise is something called “skull crushers”. You do not understand what this means, but you proceed with the physical examination regardless. On exam, there is a negative Cozen, Mill’s test, and Hook test. You notice significant tenderness to palpation just proximal to the olecranon, and pain with resisted elbow extension. What is the most appropriate initial step for this condition?

A Corticosteroid injection
B X-rays
C RICE
D Surgery

A

Answer: C

Explanation:
• Triceps tendonitis, i.e. acute inflammation of the triceps tendon due to repetitive overuse.
• Acutely RICE measures should be taken to cool down the pain and inflammation (rest, ice, compression, elevation).
• Once there is no longer pain at rest, physical therapy (PT) for the triceps tendon should be started to prevent worsening and recurrence of this condition, and to prevent progression to tendonosis (chronic degeneration of tendon).
• X-rays could be considered if the patient fails PT and RICE.
• Corticosteroid injections near the tendon are generally NOT indicated, as they can weaken the tendon, but could be considered before considering surgery as a last resort.

253
Q

A 36 year-old male with no prior medical history presents with gradual onset right groin pain. He denies trauma, weakness, numbness, or tingling. He is a former professional basketball player. On exam, log roll is negative. There is no tenderness to palpation about the groin. With the patient supine, flexing the hip to 90 degrees, adducting beyond midline, and internally rotating the femur reproduces the patient’s groin pain. What is the most appropriate initial step in management?

A Hip x-rays
B Hip MRI
C Physical therapy
D Orthopedic surgery consult

A

Answer: C

Explanation:
• Younger patients with significant athletics history can develop abnormal contact between the femur and acetabulum during ROM, causing groin pain during activity.
• This abnormal contact is the result of CAM and/or Pincer lesions developing on the hip joint.
• CAM lesion is essentially a large outgrowth of bone at the head/neck region of the femur.
• Pincer lesion is an outgrowth of bone on the acetabulum.
• Either can cause abnormal impingement of the hip during range of motion.
• Physical exam maneuvers for FAI include anterior and lateral hip impingement tests.
○ The anterior hip impingement test (described in this question) can reproduce FAI and the patient’s pain.
○ The lateral hip impingement test is performed with the hip in abduction, flexion, and external rotation, and may also reproduce the patient’s pain in FAI
• Labral tears often occur secondary to FAI.
• Treatment involves trial of physical therapy.
• If this fails, consider hip x-rays with concern for FAI.
• MRI can be useful to further outline the FAI pathology, and to identify labral tears (MRI arthrogram showing dye flowing into the labral defect).
• PRP or prolotherapy, or other regenerative injections can be considered in cases of FAI.
• Potentially corticosteroid injection could be trialed.
• Orthopedic surgery is indicated if conservative measures and/or injections fail.

254
Q

The most effective nonsurgical treatment for de Quervain’s tenosynovitis is:

A. Relative rest and anti-inflammatories
B. Splinting and ice packs 4 times a day
C. Corticosteroid injection and splinting
D. Corticosteroid injection alone

A

Answer: D

Local corticosteroid injection is proven effective as a treatment for de Quervain’s tenosynovitis, both with and without splinting. Injection alone produced an 83% cure rate, with injection plus splinting producing a 61% cure rate. Splinting alone produced a 14% cure rate, and rest and anti-inflammatories were of no benefit.